ÇIKMIŞ SORULAR Flashcards

1
Q

In which of the following options, the sequence leading to gastric cancer pathogenesis is in correct order?

A-normal mucosa- chronic active gastritis-atrophic gastritis-metaplasia-dysplasia-cancer

b-normal mucosa-metaplasia-active gastritis-atrophic gastritisdysplasia-cancer

c-normal mucosa- chronic active gastritis-metaplasia-atrophic
gastritis-dysplasia-cancer

d-normal mucosa-atrophic gastritis- chronic active gastritis-metaplasia-dysplasia-cancer

A

A-normal mucosa- chronic active gastritis-atrophic gastritismetaplasia-dysplasia-cancer

How well did you know this?
1
Not at all
2
3
4
5
Perfectly
2
Q

Which of the following cell types first arrive at the site of the injury?

Monocytes
Thrombocytes
Neutrophils
Fibroblasts
Macrophages

A

Thrombocytes

How well did you know this?
1
Not at all
2
3
4
5
Perfectly
3
Q

I- Arteria Colica Media
II- Arteria Colica Sinistra
III- Arteria Sigmoidea
IV- Arteria Rectalis Superior
V- Arteria Ileocolica

Which of the following contains the branches of the inferior mesenteric artery?

A

II, III and IV

How well did you know this?
1
Not at all
2
3
4
5
Perfectly
4
Q

Falciform ligament of the liver divides:

The right lobe into anterior and posterior segments
The left lobe into medial and lateral segments
The liver into right and left lobes
The caudate lobe into medial and lateral segments

A

The left lobe into medial and lateral segments

How well did you know this?
1
Not at all
2
3
4
5
Perfectly
5
Q

Which of the following does not play a role in the pathophysiology of inguinal hernia?

Low birth weight
Causes that increase intraabdominal pressure
Collagen synthesis disorder
Hydrocele
Patency of the processus vaginalis

A

Hydrocele

How well did you know this?
1
Not at all
2
3
4
5
Perfectly
6
Q

A metastatic mass with a diameter of 4 cm in the right liver lobe was detected in the abdominal PET of a 60-year-old male patient with esophageal cancer.
Which of the following is the patient’s stage?

T1N0M0
T0N0M1
T2N2M0
T3N0M0
T3N2M1

A

T3N2M1

Not: Since there is distant metastasis, it must be M1 independent of T and N. In addition, since there is a tumor of 4 cm in diameter the T stage cannot be zero

How well did you know this?
1
Not at all
2
3
4
5
Perfectly
7
Q

A 34-year-old previously healthy male patient notices that his eyes are yellow. He has a history of sleeve gastrectomy due to morbid obesity 16 months ago. On physical examination he has jaundice and scleral icterus along with right upper quadrant tenderness. His temperature is 38.9. Ultrasound of the abdomen demonstrates biliary ductal dilation with gallstones. Which of the following is the most appropriate treatment option for this patient?

1- Surgical Removal of the gallstones
2- Endoscopic retrograde cholangiopancreatography(ERCP) and antibiotics
3- Laparoscopic cholecystectomy
4- Observation with repeat ultrasound examinations
5- Percutaneus drainage of gallbladder

A

2- Endoscopic retrograde cholangiopancreatography(ERCP) and antibiotics

Not: Koledok taşına bağlı kolanjit nedeniyle ERCP ile taş çıkarılıp, antibiyotik verilmesi gerekir

How well did you know this?
1
Not at all
2
3
4
5
Perfectly
8
Q

Which one is not a complication of hydatid disease in humans?

Anaphylaxis
Chronic blood loss and anemia
Liver abscess
Cyst development within the liver or lungs

A

Chronic blood loss and anemia

Not: Hydatid disease may cause; Anaphylaxis due to rupture of the cyst, liver abscess due to infection of the cyst within the liver and cyst development may ocur anywhere in the body most often within the liver. Taenia type of tapeworm infections may cause gastrointestinal symptoms.

How well did you know this?
1
Not at all
2
3
4
5
Perfectly
9
Q

In the context of uncomplicated diverticulitis, what imaging modality is often used for confirmation of diagnosis and evaluation of severity?

Computed tomography (CT) scan
Magnetic resonance imaging (MRI)
Abdominal X-ray
Colonoscopy

A

Computed tomography (CT) scan

Not: CT imaging can identify signs of inflammation, such
as wall thickening and pericolonic fat stranding

How well did you know this?
1
Not at all
2
3
4
5
Perfectly
10
Q

Which of the following conditions is characterized by the presence of dilated, tortuous veins in the submucosa of the rectum and anus, often associated with painless rectal bleeding?

Anal fissure
Anal fistula
Hemorrhoids
Anal abscess

A

Hemorrhoids

How well did you know this?
1
Not at all
2
3
4
5
Perfectly
11
Q

Which of the following is not found in idiopathic granulomatous lobular mastitis pathology?

Epithelioid cell granuloma
Langhans giant cell
Lymphohistiocytic aggregate
Caseous granulomas

A

Caseous granulomas.

Found in tuberculosis

How well did you know this?
1
Not at all
2
3
4
5
Perfectly
12
Q

Which of the following laboratory findings is more suggestive of common bile duct obstruction rather than cholecystitis?

Elevated alkaline phosphatase
Elevated serum lipase
Elevated direct bilirubin
Elevated serum amylase

A

Elevated direct bilirubin

How well did you know this?
1
Not at all
2
3
4
5
Perfectly
13
Q

Which of the following is a key component of the initial management of a patient with suspected intestinal obstruction?

Immediate surgery
Oral fluid intake
Bowel rest and nasogastric decompression
Intravenous antibiotics

A

Bowel rest and nasogastric decompression

Not: This helps relieve distension, reduce vomiting, and prevent aspiration. Immediate surgery is reserved for cases of strangulation or failure of conservative measures. Oral fluid intake is generally restricted until the obstruction is resolved

How well did you know this?
1
Not at all
2
3
4
5
Perfectly
14
Q

What is the primary surgical treatment for early-stage cholangiocarcinoma involving the common bile duct?

Endoscopic stent placement
Whipple procedure
Extrahepatic bile duct resection
Liver transplantation

A

Extrahepatic bile duct resection

Not: This involves removal of the affected segment of the bile duct along with regional lymph nodes

How well did you know this?
1
Not at all
2
3
4
5
Perfectly
15
Q

Which of the following treatment modalities has the highest survival rate for multifocal hepatocellular carcinoma on the basis of chronic viral hepatitis?

Chemoembolisation combined with adjuvant chemotherapy
Antiviral therapy
Liver transplantation
Liver resection surgery
Chemoembolisation combined with adjuvant chemotherapy

A

Liver transplantation

How well did you know this?
1
Not at all
2
3
4
5
Perfectly
16
Q

Which of the following special hernia types is associated with inflamed appendix vermiformis in the hernia sac

Petit
Amyand
Littre
Richter
Maydl

A

Amyand

How well did you know this?
1
Not at all
2
3
4
5
Perfectly
17
Q

Which of the following is not an indication for secondary wound debridement?

Presence of devitalized fat tissue
Aim of achieving cosmetically better results
Presence of tense wound edges
Presence of narrow pedicule skin flaps

A

Presence of tense wound edges

How well did you know this?
1
Not at all
2
3
4
5
Perfectly
18
Q

Loop diuretics exert their effects at which part of the nephron?

Proximal convoluted tubule
ADH receptor
Distal convoluted tubule
Thick ascending limb of the loop of Henle
Collecting ducts

A

Thick ascending limb of the loop of Henle

How well did you know this?
1
Not at all
2
3
4
5
Perfectly
19
Q

What is the classic triad of symptoms associated with chronic pancreatitis?

Hematemesis, melena, and anemia
Abdominal pain, weight loss, and diarrhea
Fever, abdominal distension, and constipation
Jaundice, dark urine, and pale stools

A

Abdominal pain, weight loss, and diarrhea

Not: The classic triad of symptoms in chronic pancreatitis includes abdominal pain (often epigastric and radiating to the back), weight loss, and diarrhea. These symptoms result from the impaired digestive and endocrine functions of the pancreas.

How well did you know this?
1
Not at all
2
3
4
5
Perfectly
20
Q

Which of the following is incorrect for visceral pain?

It is primarily carried by C type nerve fibers.
It is often felt in the midline.
It does not project to cerebral cortex.
It can be reproduced with physical examination.

A

It can be reproduced with physical examination.

Not: The visceral pain transmission end in thalamus and is carried by central nervous system. Thus visceral pain is constant and can’t be reproduced with physical examination.

How well did you know this?
1
Not at all
2
3
4
5
Perfectly
21
Q

Ketamine is used occasionally as an induction anesthetic. Which of the following statements about ketamine is false?

*Ketamine increases the heart rate.
*Upon waking after receiving ketamine, patients may experience emergence phenomena.
*In addition to its induction effects, ketamine acts as a painkiller, so fewer opioids have to be used to prevent
excessive sedation.
*Ketamine decreases blood pressure.

A

*Ketamine decreases blood pressure.

How well did you know this?
1
Not at all
2
3
4
5
Perfectly
22
Q

Which of the following is a proliferative breast lesion without atypia?

Lobular carcinoma in situ
Fat necrosis
Galactocele
Sclerosing adenosis

A

Sclerosing adenosis

Not: Galactocel is a nonproliferative breast lesion.
Lobular carcinoma in situ is a proliferative breast lesion with atypia. Fat necrosis is another benign breast lesion.

How well did you know this?
1
Not at all
2
3
4
5
Perfectly
23
Q

Which of the following is most likely to present with Coruvoisier’s sign?

Distal pancreatic tumor
Gallbladder cancer
Klatskin’s tumor
Periampullary tumor

A

Periampullary tumor

Courvoisier’s principle (known as Courvoisier’s
sign or Courvoisier–Terrier’s sign, or Courvoisier syndrome)
states that a painless palpably enlarged gallbladder that is
found on physical examination. The mechanism is the slow
and steady enlargement of gallbladder due to chronic

How well did you know this?
1
Not at all
2
3
4
5
Perfectly
24
Q

What are the components of the Courvoisier-Terrier finding?

Gallstone diverticulosis hiatus hernia
Melana obstructive icterus biliary colic
Jaundice fever
Non-sensitive hydropic gall bladder and jaundice
None

A

Non-sensitive hydropic gall bladder and jaundice

How well did you know this?
1
Not at all
2
3
4
5
Perfectly
25
Q

What is the primary determinant of plasma osmolality?

Potassium
Sodium
Proteins
Urea
Glucose

A

Sodium

How well did you know this?
1
Not at all
2
3
4
5
Perfectly
26
Q

A child has a panic attack and breathes rapidly. Which of the following should we expect?

Normal acid-base balance
Metabolic alkalosis
Metabolic acidosis
Respiratory alkalosis
Respiratory acidosis

A

Respiratory alkalosis

How well did you know this?
1
Not at all
2
3
4
5
Perfectly
27
Q

Which of the following is not used for the treatment of methyl alcohol intoxication?

Thiamine
Protamine
Hemodialysis
Ethanol
Fomepizole

A

Protamine

Not: An appropriate approach for the treatment of methyl alcohol intoxication is to administer thiamine, folate, and multivitamin supplements, folinic acid 50 mg iv every 4 - 6 hours for 24 hours, in methanol ingestion to provide the cofactor for formic acid elimination CO2 and H2o, Administer ethanol or fomepizole to inhibit the metabolism of ethylene glycol or methanol, and hemodialysis for visual impairment, renal failure, pulmonary edema, and acidosis

How well did you know this?
1
Not at all
2
3
4
5
Perfectly
28
Q

A patient presents with diverticular bleeding, with hematochezia and stable vital signs. What is the appropriate initial management for this patient?

Transcatheter arterial embolization
Sclerotherapy
Observation and supportive care
Emergent surgical intervention

A

Observation and supportive care

Not: For stable patients with diverticular bleeding and no active hemorrhage, the appropriate initial management is
observation and supportive care. This may include
intravenous fluids and blood transfusion if necessary.

How well did you know this?
1
Not at all
2
3
4
5
Perfectly
29
Q

What is the first-line treatment of anaphylaxis?

Epinephrine and IV fluids
Broad-spectrum antibiotics and IV fluids
Corticosteroids and antihistamines
Beta-agonists and IV fluids
Corticosteroids and IV fluids

A

Epinephrine and IV fluids

How well did you know this?
1
Not at all
2
3
4
5
Perfectly
30
Q

A 72-year-old male was admitted with left lower abdominal pain for the past 2 days. He also complains of nausea and vomiting. His past medical history is notable for a history of chronic constipation, however, he takes no medications. His temperature is 38.3°C, blood pressure is 120/75 mmHg, and pulse is 92 beats per minute. He has left lower quadrant tenderness with palpation and no flank tenderness. Labs are collected and WBC: 16,000/mm³; CRP: 8 mg/dL. An abdominal CT scan is performed and reveals acute diverticulitis with a 3 cm sized pelvic abscess. Which of the following should be the initial treatment plan for this patient?

Inpatient treatment with percutaneous drainage
Outpatient treatment with antibiotics
Inpatient treatment with IV antibiotics
Surgery

A

Inpatient treatment with IV antibiotics

NOT: Antibiotics are the first-line treatment for all diverticular abscesses. Percutaneous drainage may be added, if feasible, for abscesses ≥4 cm, those that do not resolve with antibiotic therapy, or in the presence of clinical deterioration.

How well did you know this?
1
Not at all
2
3
4
5
Perfectly
31
Q

What is the most appropriate next step in the management for Echinococcus granulosus positive patients?,

Perform surgical resection of the cyst with a goal of complete excision

Refer the patient for percutaneous drainage and albendazole therapy

Initiate watchful waiting and monitor the cyst by imaging every 6 months

Administer albendazole therapy for 3 months

A

refer the patient for percutaneous drainage and
albendazole therapy

How well did you know this?
1
Not at all
2
3
4
5
Perfectly
32
Q

In acute pancreatitis, which imaging modality is preferred for confirming the diagnosis?

Ultrasound, MRI, X-ray, CT scan, MRI, X-ray

A

CT scan

How well did you know this?
1
Not at all
2
3
4
5
Perfectly
33
Q

Which of the following is not necessarily considered in the initial evaluation of a fresh wound?

Deep tissue damage
Presence of possible foreign bodies
Presence of devitalised tissue
Microbial contamination
Active bleeding

A

Microbial contamination

Not: Although microbial contamination might be a worrisome issue in the healing process of a wound, it is not considered during the initial evaluation of a fresh wound for many practical reasons:
1. Wound or surgical site infection does not become a clinical issue before 72 hours
2. It is impractical and unnecessary to obtain routine wound
cultures as the wound might probably be contaminated with skin flora and the treatment (or closure) of the wound should not be delayed until culture results arrive.
3. Antibiotics usually suffice to treat uncomplicated wound
infections if need be.

How well did you know this?
1
Not at all
2
3
4
5
Perfectly
34
Q

In which of the following diagnostic laparoscopy is not performed?

Adrenal tumor
Appendicitis
Endometriosis
Ectopic pregnancy

A

Adrenal tumor

How well did you know this?
1
Not at all
2
3
4
5
Perfectly
35
Q

Which of the following features belongs to intestinal type gastric cancer?

Familial tendency
Poorly differentiated, signet ring cells
Decreased E-cadherin
Increasing incidence with age
Transmural, lymphatic spread

A

Increasing incidence with age

Not: The incidence of intestinal type gastric cancer is more
common in older age group, while diffuse type is encountered more in younger age groups.

How well did you know this?
1
Not at all
2
3
4
5
Perfectly
36
Q

What blood product has the shortest shelf life?

Cryoprecipitate
Fresh frozen plasma
Packed red cells
Whole blood
Platelets

A

Platelets

How well did you know this?
1
Not at all
2
3
4
5
Perfectly
37
Q

What is the cause of complex regional pain syndrome?

Peripheral sensitization
Autoimmune activity
Genetic predisposition
Inflammatory response
Post-infection complication

A

Peripheral sensitization

How well did you know this?
1
Not at all
2
3
4
5
Perfectly
38
Q

Which of the following is the most common cause of acute mesenteric ischemia?

Acute mesenteric artery embolism
Non-occlusive mesenteric ischemia
Acute mesenteric artery thrombosis
Acute mesenteric vein thrombosis

A

Acute mesenteric artery embolism

How well did you know this?
1
Not at all
2
3
4
5
Perfectly
39
Q

Which of the following interventions is most appropriate in the initial management of a hypovolemic shock patient?

Rapid administration of crystalloid fluids
Blood transfusion to restore oxygen-carrying capacity
Administration of a beta-blocker
Application of warm blankets to maintain normothermia

A

Rapid administration of crystalloid fluids

Not: In hypovolemic shock, the primary goal is to restore intravascular volume and improve tissue perfusion. Rapid administration of crystalloid fluids, such as normal saline or lactated Ringer’s solution, helps achieve this by replenishing circulating blood.

How well did you know this?
1
Not at all
2
3
4
5
Perfectly
40
Q

Which type of the hiatal hernias are not expected to cause gastroesophageal reflux?

type 1 2 3 4 5

A

type 2

In pure type 2 hiatal hernias, gastroesophageal junction is in
regular position,

How well did you know this?
1
Not at all
2
3
4
5
Perfectly
41
Q

Which of the following sign suggests a more severe, potentially ischemic, intestinal obstruction that may require urgent surgical intervention?

Abdominal distension
Rebound tenderness on abdominal examination
Passage of flatus
High-pitched bowel sounds
Abdominal distension

A

Rebound tenderness on abdominal examination

Not: Abdominal distension, high-pitched bowel sounds, and the passage of flatus are signs that may be present in both simple and severe obstruction.

How well did you know this?
1
Not at all
2
3
4
5
Perfectly
42
Q

Pheochromocytoma is known as a “10% tumor”. Which of the following findings support the 10% rule?

  1. 10% are familial.
  2. 10% are extra-adrenal.
  3. 10% are bilateral.
  4. 10% are malignant.
  5. 10% occur in children.
A

All.

How well did you know this?
1
Not at all
2
3
4
5
Perfectly
43
Q

Which of the following symptoms for gastric cancer is not true?

Dyspnea may develop in gastric cancer patients with pleural effusions.

Jaundice may be encountered in advanced staged gastric cancer.

Anorexia and weight loss is common in gastric cancer patients.,

Patients usually feel discomfort after meals.

Dysphagia is common in gastric tumors located in pylorus.

A

Dysphagia is common in gastric tumors located in pylorus.
this statement is incorrect.

Not: Dysphagia is common in cancers located at gastric cardia. The tumors located in pylorus generally cause vomiting secondary to gastric outlet obstruction

How well did you know this?
1
Not at all
2
3
4
5
Perfectly
44
Q

A patient with cirrhosis develops spontaneous bacterial peritonitis (SBP). What is the most common organism responsible for SBP in this population?

Streptococcus pneumoniae
Klebsiella pneumoniae
Staphylococcus aureus
Escherichia coli

A

Escherichia coli

Not: Streptococcus pneumoniae is more commonly associated with community-acquired pneumonia. Klebsiella pneumoniae can cause liver abscesses but is less common in SBP. Staphylococcus aureus is not a typical cause of SBP.

How well did you know this?
1
Not at all
2
3
4
5
Perfectly
45
Q

A 32-year-old woman, is brought to the emergency room after a motor vehicle accident. She was the driver and was unrestrained during the collision. On examination, she complains of severe abdominal pain. Vital signs are stable, but there is tenderness and bruising over her abdomen.
As you assess her, you notice her abdomen is distended.
Which option is the most appropriate initial step in the management of this patient’s abdominal trauma?

*Immediately proceed to surgical exploration given the
mechanism of injury and clinical findings.

*Perform a chest X-ray to rule out associated thoracic injuries before deciding on further abdominal imaging.

*Administer analgesics and observe her for a few hours, then repeat the physical examination to assess for any changes.

*Order a focused assessment with sonography for trauma
(FAST) to evaluate for intra-abdominal fluid.

A

Order a focused assessment with sonography for trauma
(FAST) to evaluate for intra-abdominal fluid.

Not: FAST is a rapid bedside ultrasound examination commonly used in the initial assessment of trauma patients. It can quickly identify the presence of free intra-abdominal fluid, such as blood, which may indicate internal organ injury or bleeding.

How well did you know this?
1
Not at all
2
3
4
5
Perfectly
46
Q

A patient with a localized pancreatic adenocarcinoma involving the head of the pancreas and adjacent structures may undergo:

Enucleation
Distal pancreatectomy
Whipple procedure
Total pancreatectomy

A

Whipple procedure AKA pancreaticoduodenectomy.
This involves the removal of the head of the pancreas, duodenum, and part of the common bile duct.

How well did you know this?
1
Not at all
2
3
4
5
Perfectly
47
Q

Papillary thyroid cancer differs from follicular thyroid cancer by:
*Mortality rate is lower in follicular thyroid cancer.
*Recurrence rate is lower in follicular thyroid cancer.
*Lymph node metastasis is more common in papillary thyroid cancer.
*Vascular invasion incidence is lower in papillary thyroid cancer.

A

Lymph node metastasis is more common in papillary
thyroid cancer.

How well did you know this?
1
Not at all
2
3
4
5
Perfectly
48
Q

Which of the following statements is true?

*Thyroglobulin is a commonly used tumor marker in the assessment of thyroid cancer.
*FNA biopsy can be diagnostic for follicular carcinoma.
*Follicular thyroid carcinoma may be associated with flushing and diarrhea.
*Papillary carcinoma is the most common and the most aggressive form of thyroid cancer.
*Anaplastic carcinoma is the least common and the most
aggressive form of thyroid cancer.

A

Anaplastic carcinoma is the least common and the most
aggressive form of thyroid cancer. This is true.

NOT: Papillary carcinoma is the most common thyroid cancer, approximately 85% of cases, the least aggressive.
Follicular carcinoma is the second most common, accounting for approximately 10% of cases, with a slightly more aggressive course. FNA cytology alone cannot distinguish between follicular adenoma and carcinoma. The diagnosis requires histologic evaluation of a surgical specimen. MTC is a neuroendocrine tumor of the C cells of the thyroid gland. Patients with advanced disease may present with flushing or diarrhea secondary to hormone secretion by the tumor.
Medullary carcinoma is the only thyroid cancer that reliably
expresses a tumor marker, calcitonin. Thyroglobulin levels may be elevated in benign thyroid disease and are not reliably elevated in follicular or papillary carcinomas, and therefore not useful as a tumor marker. Anaplastic carcinoma is the least common but most aggressive form of thyroid cancer with a dismal prognosis

How well did you know this?
1
Not at all
2
3
4
5
Perfectly
49
Q

A premenopausal 44-year-old woman undergoes a quadrantectomy and node dissection for a 2-cm infiltrating carcinoma of the left breast. The margins are clear and 5 out of 15 lymph nodes are involved. ER and PR are positive. Recommended adjuvant therapy should include which of the following?

Modified radical mastectomy
Chemotherapy, radiotherapy, and tamoxifen.
Radiotherapy alone
Estrogen therapy alone
Chemotherapy alone

A

Chemotherapy, radiotherapy, and tamoxifen.

Not: It is advised chemotherapy for all invasive cancers >1 cm as well as for node-positive cancers. Radiotherapy is required whenever breast conserving surgery is undertaken, and tamoxifen should be given for all ER+ and/or PR+ invasive tumors whose patients are premenopausal.

How well did you know this?
1
Not at all
2
3
4
5
Perfectly
50
Q

What is the objective finding to tell a wound is in proliferation phase?

Synthesis of Platelet Derived Growth Factor in the wound
Proliferation of fibroblasts in the wound
Synthesis of collagen in the wound
Presence of Macrophages in the wound
Presence of T cells in the wound

A

Presence of T cells in the wound

Not: T cells appear in the wound about a week after the injury and their presence marks the starting of the proliferation phase of the wound healing.

How well did you know this?
1
Not at all
2
3
4
5
Perfectly
51
Q

46-y.o. patient underwent surgery because of a prolapsed intervertebral disk. General anesthesia was induced with propofol and maintained with sevoflurane. When administration of sevoflurane was discontinued, the patient regained consciousness in a few minutes. Which of the following statements best explains why anesthetic recovery is so rapid when sevoflurane administration is stopped?

It is rapidly metabolized within the CNS.
It has a low minimum alveolar concentration (MAC) value.
It has a low blood/gas partition coefficient.
It distributes mainly into the cerebral cortex.

A

It has a low blood/gas partition coefficient.

How well did you know this?
1
Not at all
2
3
4
5
Perfectly
52
Q

A patient is in anaphylactic shock. The patient is given Epinephrine intramuscularly. This medication will have what effect on the body?

It will cause vasoconstriction and decrease the blood pressure.
It will help dilate the airways.
It will help block the effects of histamine in the body.
It will prevent a recurrent attack

A

It will help dilate the airways.

Not: Epinephrine acts as a vasopressor and will actually dilate the airway. Epinephrine performs vasoconstriction which will INCREASE the blood pressure. It does not prevent a recurrent attack (corticosteroids may help with this), and it does not block the effects of histamine (antihistamine helps with this).

How well did you know this?
1
Not at all
2
3
4
5
Perfectly
53
Q

Which of the following is the type of diaphragmatic hernia that occurs more frequently in people around the age of 40 and is sometimes characterized by bleeding, acute gastric dilatation and volvulus?

Paraesophageal Hernia
Sliding Hernia
Morgagni Hernia
Bochdalek Hernia
Traumatic Diaphragmatic Hernia

A

Paraesophageal Hernia

Not: Paraesophageal hernia is more often asymptomatic but the occurrence of gastric volvulus and bleeding is more frequent than other diaphragmatic hernias

How well did you know this?
1
Not at all
2
3
4
5
Perfectly
54
Q

Which of the following is the type of polyp in Peutz Jeghers Syndrome?

Adenomatous
Hamartomatous
Benign Lymhoid
Inflammatory
Hyperplastic

A

Hamartomatous

How well did you know this?
1
Not at all
2
3
4
5
Perfectly
55
Q

Which of the following does not cause Courvoisier’s law ?

Pancreatic cancer
Papilla tumor
Klatskin tumor
Distal bile duct cancer
Cholangiocarcinoma of the distal common bile duct
Carcinoma of the second segment of duodenum
Pancreatic head carcinoma

A

Klatskin tumor

Not: Klatskin tumor will not cause Courvoisier’s law as the obstruction is above the junction of cystic duct and main hepatic duct

How well did you know this?
1
Not at all
2
3
4
5
Perfectly
56
Q

Fastest induction and recovery is seen with -

Enflurane
N2O
Halothane
Desflurane

A

Desflurane

Not: Due to its low blood gas partition coefficient (0.42), desflurane provides fast induction and recovery in high flow anesthesia.

How well did you know this?
1
Not at all
2
3
4
5
Perfectly
57
Q

Which of the following target points is not correct for a trauma patient needing transfusion of blood products?

*Cryoprecipitate should be administered to the patients with less than 100 m g/dL of fibrinogen.
*Fresh frozen plasma should be administered to the patients with INR more than 1.5
*Transfusion of packed red blood cells should be continued until hemoglobin levels reach at least 10 g/dL.
*Packed red blood cells should be administered to the patients with less than 7 g/dL of hemoglobin.
*Thrombocytes should be administered to the patients with platelet counts less than 150,000/µL.

A

Thrombocytes should be administered to the patients with platelet counts less than 150,000/µL.

NOT: The minimum required amount of platelet count for hemostasis is 50000/µL. 150000/µL. Is more than enough and is not a necessary end point.

How well did you know this?
1
Not at all
2
3
4
5
Perfectly
58
Q

The most common cause of an acute hemolytic transfusion reaction is:

Defective blood filter
An error during type and crossmatch
An error during type and screen
Misidentification of the patient, blood specimen, or transfusion unit

A

Misidentification of the patient, blood specimen, or transfusion unit

Not: Acute hemolytic transfusion reactions are usually due to ABO blood incompatibility, and the most common cause is misidentification of the patient, blood specimen, or transfusion unit (clerical error).

How well did you know this?
1
Not at all
2
3
4
5
Perfectly
59
Q

Which of following signs is included in the qSOFA organ failure assessmant?

Tachycardia
Hypertansion
Bradicardia
Altered mental status

A

Altered mental status

Not:Quick Sequential Organ Failure Assessment (qSOFA) identified hypotension, altered mental status and tachpnea as harbingars of a poor sepsis outcome.

How well did you know this?
1
Not at all
2
3
4
5
Perfectly
60
Q

Which of the following are the cells in the gastric epithelium that secrete HCL?

G cells
Chief cells
Parietal cells
Foveolar cells
Mast cells

A

Parietal cells

How well did you know this?
1
Not at all
2
3
4
5
Perfectly
61
Q

Which of the following entities endoscopic ultrasonography (EUS) is incapable of detecting?

Location of the tumor
The depth of the tumor
Local lymph node involvement
Size of the tumor
The presence of distant metastasis

A

The presence of distant metastasis

How well did you know this?
1
Not at all
2
3
4
5
Perfectly
62
Q

Which of the following is not expected to develop in chronic pancreatitis?

Secondary diabetes
Pancreatic duct obstruction
Systemic organ failure
Pseudocyst
Malabsorption

A

Systemic organ failure

Not: development of systemic organ failure is expected in acute pancreatitis, this risk is absent in chronic pancreatitis.

How well did you know this?
1
Not at all
2
3
4
5
Perfectly
63
Q

Which of the following is not observed in acute pancreatitis?

Malabsorption
Acute kidney failure
Pancreatic pseudocyst
Liquid electrolyte imbalance
Pancreatic abscess

A

Acute kidney failure

How well did you know this?
1
Not at all
2
3
4
5
Perfectly
64
Q

A 22 year old woman presents with a painful fluctuant mass in the midline between the gluteal folds. She denies pain on rectal examination. Which of the following is the most likely diagnosis?

Fistula
Perirectal abscess
Anal fissure
Pilonidal abscess
Perianal abscess

A

Pilonidal abscess

Not: There is absce ss in the intergluteal sulcus and since the anal examination is normal, most likely diagnosis is pilonidal cyst with an abscess

How well did you know this?
1
Not at all
2
3
4
5
Perfectly
65
Q

A 28-year-old female presents several weeks after having sustained an injury to her left breast. She has a painful mass in the upper outer quadrant. Skin retraction and ecchymosis is noticed and a hard mass, 3–4 cm in diameter, can easily be palpated. What is the most likely diagnosis?

Fat necrosis and hematoma
Infiltrating carcinoma
Sclerosing adenosis
Breast abscess
Fibroadenoma

A

Fat necrosis and hematoma

Not: Sclerosing adenosis is a variant of fibrocystic disease
and may present with a hard mass. In a hematoma, evidence of resolving ecchymosis may be present.

How well did you know this?
1
Not at all
2
3
4
5
Perfectly
66
Q

Which one of the following is true regarding the hydatid disease of the liver in humans?

The disease occurs by ingesting eggs by eating sheep liver
The disease occurs by ingesting eggs from direct contact to dogs
The disease occurs by ingesting eggs from vegetables which are contaminated by the dogs’ feces
All of the above

A

The disease occurs by ingesting eggs from vegetables which are contaminated by the dogs’ feces

How well did you know this?
1
Not at all
2
3
4
5
Perfectly
67
Q

Galactorrhea, a milky discharge from the nipple in nonpregnant women, is most likely to be associated with which of the following?

Fibroadenoma
Tubular adenoma
Breast abscess
Hyperparathyroidism
Pituitary microadenoma

A

Pituitary microadenoma

Not: Galactorrhea is fairly common up to old age. The discharge may vary in color from brown to milky. Hormonal causes are associated with elevated prolactin levels or with pituitary or thyroid disorders. Tranquilizers have also been implicated. Simple abscesses do not cause galactorrhea.

How well did you know this?
1
Not at all
2
3
4
5
Perfectly
68
Q

A 36-year-old woman complains of a 3-month history of bloody discharge from the nipple. At examination, a small nodule is found, deep to the areola. Careful palpation of the nipple areolar complex results in blood appearing at the 3 O’clock position. Mammogram findings are normal. What is the likeliest diagnosis?

Carcinoma in situ
Fat necrosis
Breast cyst
Intraductal papilloma
Intraductal carcinoma

A

Intraductal papilloma

NOT: Intraductal papilloma is the most common cause of bloody discharge from the nipple. The lesion is treated by excision and is benign in most cases. Cancer is present in 5% of cases. Preoperative ductography can be used to help locate the offending duct.

How well did you know this?
1
Not at all
2
3
4
5
Perfectly
69
Q

Which of the following is one of the goals to be achieved for successful shock therapy?

Central venous oxygen saturation of 50-60%
35-40% mixed venous oxygen saturation
Mean arterial pressure > 65 mmHg
Cardiac index 1.5 TL / minute / m2
lactate level > 4 mMol / L

A

Mean arterial pressure > 65 mmHg

NOT: All other situations are parameters that show the severity of the shock increased

How well did you know this?
1
Not at all
2
3
4
5
Perfectly
70
Q

In which clinical setting urgent appendectomy is not indicated?

Perforated Appendicitis with Localized Peritonitis
Phlegmonous (Plastrone) Appendicitis
Acute Appendicitis
Perforated Appendicitis with Generalized Peritonitis
Acute Appendicitis in 1st Trimester Pregnancy

A

Phlegmonous (Plastrone) Appendicitis

NOT: Phlegmonous (Plastrone) appendicitis, an appendectomy is performed 4-6 weeks later in a planned manner

How well did you know this?
1
Not at all
2
3
4
5
Perfectly
71
Q

A 20-year-old woman is seen in clinic because of a thyroid nodule. She is asymptomatic and her past medical history is unremarkable. She takes no medications. There is a 1-cm firm, solitary, nodule in the lateral aspect of the left lobe of the thyroid. A radionuclide scan showed no uptake of tracer in the nodule. Ultrasonography shows a solid, homogeneous 1-cm mass. FNA cytology shows a follicular neoplasm. What is the next best step in management?

Total thyroidectomy
Thyroid suppression with thyroxine
Left thyroid lobectomy
Incisional biopsy and enucleation if benign
Irradiation (radioactive iodine)

A

Left thyroid lobectomy

NOT: y. Even though an FNA showing follicular cells is only 5% likely to be a malignancy, most endocrine surgeons would recommend excision because of that concern.

How well did you know this?
1
Not at all
2
3
4
5
Perfectly
72
Q

Which one is wrong about rhythms that can cause cardiac arrest?

+Biphasic: 120 – 360 J , monophasic: 360 J shock is applied during Pulse-less VT
+Pulse-less electrical activity is not shockable rhythm
+Amiadarone can be used as post-shock anti-arrhythmic agent during CPR
+Monophasic 360J is used while asystole shock is applied
+Ventricular fibrillation is shockable rhythm

A

Monophasic 360J is used while asystole shock is applied, this statement is wrong.

Not: 1. Shockable rhythms:
* Ventricular fibrillation (Biphasic: 120 – 360 J , monophasic:
360 J)
* Pulse-less ventricular tachycardia (Biphasic: 120 – 360 J ,
monophasic: 360 J)
2. Non-shockable rhythms:
* ASYSTOLE
* Pulse-less electrical activity

How well did you know this?
1
Not at all
2
3
4
5
Perfectly
73
Q

75 yo male admitted to ER due to sudden and very severe epigastric pain felt immediately after eating. He has a
history of oral anticoagulant use due to heart valve insufficiency. Initial physical examination shows no pathological findings except for minimal distention and
tenderness in the abdomen. In the laboratory tests, there is an increase D-Dimer and there are signs of metabolic acidosis. What is your possible pre-diagnosis?

Colon tumor
Acute gastroenteritis
Acute gallstone cholecystitis
Acute appendicitis
Ischemia due to acute mesenteric artery embolism

A

Ischemia due to acute mesenteric artery embolism

NOT: AMI is characterized by advanced age, valvular diseases, sudden abdominal pain, mild abdominal findings in the initial examination, and increased blood ischemia markers.

How well did you know this?
1
Not at all
2
3
4
5
Perfectly
74
Q

Which of the below eponyms is the definition of edema and ecchymosis around the umblical area which can be encountered in acute pancreatitis that was first defined
by a gynecologist in patients with ruptured ectopic pregnancy?

Caput medusa
Cullen’s sign
Grey - Turner’s sign
Murphy’s sign
Clepotage

A

Cullen’s sign

NOT: Cullen’s sign is superficial edema and bruising in the subcutaneous fatty tissue around the umbilicus.

How well did you know this?
1
Not at all
2
3
4
5
Perfectly
75
Q

An 18 yo man is rushed into the ER by his friends who left him immediately before they could interviewed by the staff. He is semiconscious RR=8 /min, BP = 120/70, pulse = 60 bpm. He is noted to have needle track marks on his arms and his pupils are small. What is the single best initial tx?

Methadone
Gastric lavage
Naloxone
Insulin

A

Naloxone

NOT: The key is naloxone. The diagnosis is opiat overdose. Points in favour are : reduced consiousness, RR 8 /min, hypotension, miosis, needle track marks on his arms

How well did you know this?
1
Not at all
2
3
4
5
Perfectly
76
Q

Which of the following is not true for primary peritonitis?

+Acute abdominal findings and positive paracentesis findings are diagnostic.
+In adults, it is most common in patients with liver cirrhosis.
+The treatment of primary peritonitis is medical.
+E.Coli, one of the enteric bacteria, is most frequently encountered pathogen in adults.
+Primary peritonitis is of multibacterial origin

A

+Primary peritonitis is of multibacterial origin

Note: It is monobacterial in origin.

How well did you know this?
1
Not at all
2
3
4
5
Perfectly
77
Q

MHC Class II molecules are a class of major histocompatibility complex (MHC) molecules normally found only in what type of cells?

In immune system cells
In all of the cells
In all of the nucleated cells
In hepatocytes

A

In immune system cells

NOT: The human major histocompatibility complex (MHC), commonly called the human leukocyte antigens (HLA) are divided into two classes. Class I antigens are expressed by all of the nucleated cells, while class II antigens are expressed by the immune system cells,

How well did you know this?
1
Not at all
2
3
4
5
Perfectly
78
Q

A 35-year-old woman is seen in clinic because of weight gain and abnormal hair growth. She has gained 15 kg in 6 months, most notably in her torso. She denies increased appetite and has not changed her daily activity patterns. She has been emotionally labile, and her previously regular menses have become irregular (periods are shorter or missed altogether). On examination, she has truncal obesity and hirsutism. The most likely primary cause of her symptoms is due to hyperfunction of which one of the following?

Pulmonary enterochromaffin (Kulchitsky) cells
Ovarian epithelial cells.
Pituitary basophils
Adrenal cortical cells
Adrenal medullary cells

A

Pituitary basophils

NOT: This patient has symptoms and signs of hypercortisolism (CS). The most common cause of CS in adults is an ACTH-secreting tumor of the pituitary basophils (CD). Women in the third and fourth decades of life are the typical patients. CD accounts for 70% of cases of CS. Bronchial carcinoid tumors (arising from Kulchitsky cells) are a source of ectopic ACTH production. Ectopic ACTH syndrome causes about 15% of CS in adults. The adrenal medulla does not produce glucocorticoids; tumors of the medulla are PHEOs and produce excess catecholamines. A tumor of ovarian epithelial cells could lead to menstrual irregularities through excess sex steroid production but would not produce hypercortisolism.

How well did you know this?
1
Not at all
2
3
4
5
Perfectly
79
Q

Which of the following neurostimulators below have excitatory effect in the modulation processes of pain?

GABA
Acetylcholine
Glutamate
Glysine
Beta endorfine

A

Glutamate

NOT: Glutamate as a neurotransmitter has excitatory effect in pain modulation processes, both in peripheral and central nervous system, while other options have inhibitory effects.

How well did you know this?
1
Not at all
2
3
4
5
Perfectly
80
Q

Which of the following neurostimulators have excitatory effect in the modulation processes of pain?

Beta endorfine
GABA
Glutamate
Glysine
Acetylcholine

A

GABA

How well did you know this?
1
Not at all
2
3
4
5
Perfectly
81
Q

Etiologies associated with hypocalemia may include all of the following except?

Vitamin D deficiency
Metastatic bone lesions
Inadequate intake calcium
Renal failure

A

Metastatic bone lesions

NOT: Metastatic bone lesions are associated with hyperclcemia due to accelerated bone metabolism and relase of calcium into the serum. Renal failure, inadequate calcium intake, and vitamin D deficiency may cause hypocalcemia.

How well did you know this?
1
Not at all
2
3
4
5
Perfectly
82
Q

A young collage student is found in his dorm unconscious. He has tachyarrythmia and high fever. He also seems to be bleeding from his nose ,which on examination shows a perforation of a nasal septum. What is the most likely dx?

Marijuana OD
Heroin OD
Cocaine OD
Alcohol OD

A

Cocaine OD

NOT: The symptoms of cocain overdose : tachyarrythmia, high fever, perforation of a nasal septum, unconscious, anxiety, paranoia, tachypnoea, increased energy and talking rapidly, dilated pupils.
Also: rabdomyolysis, metabolic acidosis, convulsion.

How well did you know this?
1
Not at all
2
3
4
5
Perfectly
83
Q

Adequate of alveolar ventilation is determined by measuring:

The oxygen gradient
Cardiac output
Carbon dioxide tension
Oxygen tension
Oxygen saturation

A

Carbon dioxide tension

NOT: (Adequate of alveolar ventilation is obtained by measured the carbon
dioxide tension. Oxygen values may be affected by ventilation, but
the measure of ventilation is carbon dioxide.)

How well did you know this?
1
Not at all
2
3
4
5
Perfectly
84
Q

An 18-year-old presents with a well circumscribed 2-cm mass in her right breast. The mass is painless and has a rubbery consistency and discrete borders. It appears to move freely through the breast tissue. What is the likeliest diagnosis?

Cystosarcoma phyllodes
Fibroadenoma
Cyst
Intramammary lymph node
Carcinoma

A

Fibroadenoma

NOT: Fibroadenomas are most often found in teenage girls. They are firm in consistency, clearly defined, and very mobile. The typical feature on palpation is that they appear to move freely through the breast tissue (“breast mouse”).

How well did you know this?
1
Not at all
2
3
4
5
Perfectly
85
Q

All of the following are risk factors for obstructive sleep apnea, EXCEPT;

Age>50
Short neck
Female gender
Enlarged tonsils
Obesity

A

Female gender

NOT: The assessment of preoperative predictability for obstructive sleep apnea can be done by using the “STOP-BANG” questionnaire. (S, snoring; T, tired during daytime; O, observed for apnea during sleep; P, high blood pressure; B, BMI >35 kg/m2 ; A, age >50; N, neck circumference >40 cm; G, male gender). In addition to the questionnaire, upper airway anatomical abnormalities that increase the likelihood of obstruction are tonsillar hypertrophy, tumors of the upper airway, or facio maxillary abnormalities

How well did you know this?
1
Not at all
2
3
4
5
Perfectly
86
Q

A 45-year-old male is referred for clinic for a left-sided adrenal mass. Serum metanephrines are markedly elevated and is scheduled for a laparoscopic left adrenalectomy. Prior to proceeding to the operating room, what intervention should be performed?

*Hydration followed by beta-blockade.
*Alpha-blockade and then the addition of beta-blockade 10 days prior to surgery
*Beta-blockade until the patient is not tachycardic and then the addition of an alpha blocker
*Left-sided adrenal biopsy to rule out cancer
*No further intervention is required. Proceed with surgery

A

*Alpha-blockade and then the addition of beta-blockade 10 days prior to surgery

This patient has been diagnosed with a left-sided PHEO. It is
appropriate to proceed with surgical intervention. However, it is essential to make sure that the patient is adequately prepared for surgery with alpha blockade, usually with either phenoxybenzamine or doxazosin, prior to surgery. Once adequate blockade has been achieved, beta-blockade should be added 10 days prior to surgery along with aggressive hydration

How well did you know this?
1
Not at all
2
3
4
5
Perfectly
87
Q

A 40-year-old healthy woman is found to have a serum calcium level of 11 mg/dL during a preventive medicine visit. She is otherwise healthy and takes no medications. There is no family history of endocrine disease. Serum phosphorus is 2.4 mg/dL, and the PTH level is 90 pg/mL. Sestamibi scan shows persistent uptake in the region of the inferior lobe of the thyroid on the right. Cervical ultrasonography demonstrates a 15-mm ovoid hypoechoic solid soft-tissue mass immediately adjacent and lateral to the inferior pole of the right thyroid lobe. Which of the following is the most appropriate treatment recommendation for this patient?

Begin saline and bisphosphonates intravenously
Observation and repeat laboratory studies in 6 months
Begin daily oral furosemide
Targeted parathyroidectomy with ioPTH monitoring
Begin daily cinacalcet

A

Targeted parathyroidectomy with ioPTH monitoring

NOT: This patient has early sporadic PHP. Although she is asymptomatic, she meets consensus criterion for parathyroid operation of age < 50. She localized preoperatively to an abnormal right inferior parathyroid
gland, so she is a candidate for a targeted approach to
parathyroidectomy using a focal exploration and ioPTH monitoring. Four-gland exploration would also be an appropriate answer if this had been an option and would be the appropriate next step if the ioPTH did not decrease as expected.

How well did you know this?
1
Not at all
2
3
4
5
Perfectly
88
Q

Which of the following has the lowest risk of developing colon cancer?

Ulcerative Colitis
Gardner Syndrome
Villous Adenoma
Hamartomatous Polyp
Familial Polyposis

A

Hamartomatous Polyp

How well did you know this?
1
Not at all
2
3
4
5
Perfectly
89
Q

Aldosterone secretion in response to fluid loss will result in which on of the following electrolyte imbalance?

Hyponatremia
Hypokalemia
Hypercalsemia
Hyperkalemia
Hypernatremia

A

Hypokalemia

NOT: Aldosterone is secreted in response to fluid loss. Aldosterone causes sodium reabsorption and potassium elimination, further exacerbating hypokalemia.

How well did you know this?
1
Not at all
2
3
4
5
Perfectly
90
Q

What is the proper maximum amount of crystalloid fluid to be administered for a fluid challenge in a patient with shock?

500 cc
800 cc
1000 cc
1500 cc
2000 cc

A

2000 cc

How well did you know this?
1
Not at all
2
3
4
5
Perfectly
91
Q

What is the least amount of systolic blood pressure required for a palpable pulsation in the radial artery?

60 mmHg
70 mmHg
80 mmHg
90 mmHg
100 mmHg

A

80 mmHg

At least 80 mmHg of systolic pressure is required to properly
palpate radial pulse, 70 for femoral and 60 for carotid

How well did you know this?
1
Not at all
2
3
4
5
Perfectly
92
Q

In which of the following situations, abdominal pain does not show a sudden and sharp onset?

Hollow organ perforation
Acute appendicitis
Ectopic pregnancy rupture
Acute mesenteric ischemia after embolism
Ovarian cyst rupture

A

Acute appendicitis

NOT: In the development of acute appendicitis, visceral pain is felt first, and therefore the onset of pain is slow and blunt at the beginning

How well did you know this?
1
Not at all
2
3
4
5
Perfectly
93
Q

62-y.o. patient undergoes surgery to repair a small bowel closed loop obstruction. Which of the following general anesthetics would be contraindicated in this patient?

Ketamine
Nitrous oxide
Thiopental
Sevoflurane

A

Nitrous oxide

NOT: In closed loop obstruction, the lumen of the bowel is occluded at two points, thus preventing prograde and retrograde movements of bowel contents. Because nitrous oxide uses into the cavity more rapidly than the air (principally nitrogen) diffuses out, the gas can increase the intraluminal pressure of the obstructed bowel loop, thus
increasing the risk of gut rupture. Examples of conditions in which nitrous oxide might be hazardous include all the closed cavities filled with air, such as pneumothorax, pulmonary air cysts, and pneumocephalus.

How well did you know this?
1
Not at all
2
3
4
5
Perfectly
94
Q

You’re providing education to a patient, who has a severe peanut allergy, on how to recognize the signs and symptoms of anaphylactic shock. Select the signs and
symptoms which is NOT associated with anaphylactic shock:

Difficulty speaking
Slow heart rate
Vomiting and Nausea
Feeling dizzy
Itchy

A

Slow heart rate

NOT: Patients who are in anaphylactic shock will have signs and symptoms associated with the effects of histamine. Remember histamine affects the respiratory, cardiac, GI and skin. The patient can have the following: Respiratory: dyspnea and wheezing (bronchoconstriction), swelling of upper airways due to edema “tightness”…can’t speak, coughing, stuffy nose, watery eyes, Cardiac: tachycardia, hypotension (vasodilation)…loss of consciousness, dizzy, GI: vomiting, nausea, pain, Skin: vasodilation…red, swollen, itchy, hives

How well did you know this?
1
Not at all
2
3
4
5
Perfectly
95
Q

Which local anesthetic does not require adrenaline¿

Lignocaine
Cocaine
Procaine

A

Cocaine

NOT: (The key is cocaine. Cocaine does not require adrenaline)

How well did you know this?
1
Not at all
2
3
4
5
Perfectly
96
Q

During the transmission of a painful stimuli to the central nervous system, which of the following processes below is not employed?

Transmission
Sensitization
Perception
Modulation
Transduction

A

Sensitization

How well did you know this?
1
Not at all
2
3
4
5
Perfectly
97
Q

Which of the following factors does not contribute to the acute coagulopathy of trauma (ACOT)?

Acidosis
Hypothermia
Release of tissue factor
Release of heparin sulphate
Activation of protein C

A

Release of tissue factor

NOT: Releasing of tissue factor contributes to clot formation however remaining factors causes coagulopathy

How well did you know this?
1
Not at all
2
3
4
5
Perfectly
98
Q

Which of the following is wrong for pancreatic pseudocyst?

It occurs with the accumulation of fluid that develops after
pancreatitis.
It does not have a complete capsule and is surrounded by tissue and granulation tissue.
It may develop after acute or chronic pancreatitis.
There is a complete epithelial capsule
Persists for more than 4 weeks

A

There is a complete epithelial capsule, this statement is wrong

NOT: The pseudocyst does not have a complete capsule, it is surrounded by tissue and granulation, therefore it is called false cyst

How well did you know this?
1
Not at all
2
3
4
5
Perfectly
99
Q

In what case is a distributive shock (hyper-dynamic, or vasodilatory shock) seen?

Acute myocardial infarction
Sepsis
Renal failure
Pulmonary embolism
Severe gastroenteritis

A

Sepsis

NOT: Severe gastroenteritis causes hypovolemic shock. In acute myocardial infarction, cardiac shock will be appeared. Pulmonary embolism causes obstructive shock. In renal failure, hypervolemia, acidosis and hyperpotassemia will develop. Cardiac shock will be more prominent. Sepsis causes hyper-dynamic, distributive or vasodilatory shock.

How well did you know this?
1
Not at all
2
3
4
5
Perfectly
100
Q

I- Sigmoid Colon
II- Descending Colon
III- Small Intestine
IV- Stomach
V- Caecum and Ascending Colon
Which of the organs above may develop volvulus?

I, II and V
I, III and V
II and III
II and IV
I,III,IV and V

A

I,III,IV and V

NOT: Descending colon is partially retroperitoneal and fixed to parietal periton with Toldt Fascia

101
Q

Which of the following is classified as primary peritonitis?

Peritonitis due to colon diverticulitis.
Peritonitis due to peptic ulcer perforation.
Peritonitis due to Systemic Lupus Erythematosus (SLE)
Peritonitis due to gallbladder perforation after acute stony
cholecystitis
Peritonitis due to Meckel diverticulitis.

A

Peritonitis due to Systemic Lupus Erythematosus (SLE)

NOT: Peritonitis seen in liver cirrhosis and SLE are spontaneous peritonitis.

102
Q

A 67-year-old man presents with right upper quadrant abdominal pain and jaundice. He is afebrile with normal vital signs. On laboratory findings, he has elevated levels of bilirubin and alkaline phosphatase. Ultrasound demonstrates gallstones, normal gallbladder wall thickness, no pericholecystic fluid, and a common bile duct of 1.0 cm. What is the most appropriate diagnosis?

Choledocholithiasis
Gallstone pancreatitis
Acute cholelithiasis
Symptomatic cholelithiasis
Cholangitis

A

Choledocholithiasis

NOT: Ateş yok, kolanjit değil, bilirubin yüksek safra kanalı taşı ile uyumlu

103
Q

Which of the following is not an indication for delayed wound closure?

Excessive edema
Excessive contamination
Wounds aged more than 24 hours
Presence of necrotic tissue or debris
Active infection

A

Presence of necrotic tissue or debris

NOT: The wounds can safely be closed after removal of necrotic tissue and/or debris.

104
Q

Which of the following methods are both diagnostic and therapeutic in a patient with suspected mesenteric artery embolism or thrombosis?

CT Angio
MRI Angio
Angiography
X-Ray
USG

A

Angiography

NOT: Angiographic intervention is both a diagnostic and therapeutic tool in mesenteric artery embolism / thrombosis

105
Q

Which of the following statements is wrong about Barret Esophagus?

*Gastric acid and bile are effective in its development
*Develops in 10% of patients with gastroesophageal reflux
*Defined as the replacement of stratified squamous cells in the esophagus with an intestinal columnar cell
*Metaplastic columnar cells are more resistant to the damage caused by reflux and the development of malignancy
*The most important cause is chronic esophageal reflux

A

Metaplastic columnar cells are more resistant to the damage caused by reflux and the development of malignancy, this statement is incorrect.

NOT: Metaplastic columnar cells are more sensitive to the damage and responsible for developing esophageal adenocancer

106
Q

Female patient, aged 68, admitted to the polyclinic with abdominal pain complaints. There was a history of coronary stenting due to atherosclerotic plaque 1 year ago. In her anamnesis, she complains about epigastric and periumbilical pain that starts 20 minutes after intense meals for 6 months. She stated that the pain is alleviated but persists when she has decreased the amount of food she consumes and now she has weight loss and diarrhea added to her symptoms. What is your preliminary diagnosis for this patient whose physical examination, endoclonoscopy and abdominal USG shows no obvious pathology?

Acute pancreatitis
Obstruction due to colon tumor
Chronic mesenteric ischemia
Peptic ulcer
Pancreatic tumor

A

Chronic mesenteric ischemia

NOT: The history of the patient has a classcial pattern suggesting a chronic mesenteric ischemia or intestinal angina. Absence of pathologic findings on ohysical examination, endoscopy and ultrasound helps to exclude other organic pathologies.

107
Q

Which of the following statements about the clinical features of abdominal wall hernias is correct?

*20% of all abdominal wall hernias develop in the groin
*The majority are femoral hernias
*Indirect inguinal hernias and femoral hernias are more common in left abdominal side
*Femoral hernias are more common in men
*Indirect inguinal hernias are more common symptomatic than direct inguinal hernias

A

Indirect inguinal hernias are more common symptomatic than direct inguinal hernias, this statement is the only correct one.

NOT: Indirect hernias are more symptomatic than direct hernias because of its origin. Indirect hernias passes through the canalis inguinalis and it is very narrow than the Triangle of Hasselbach

108
Q

If normally a non-painful stimuli causes a sensation of pain in an individual , what is this condition called?

Hyperesthesia
Hyperpathy
Allodinia
Neuropathy
Hyperalgesia

A

Allodinia

109
Q

Which is not a component of Reynolds’ pentad?

Fever
Hypotension
Jaundice
Tachycardia

A

Tachycardia

The components of Reynolds’ pentad are: Fever, right upper
qudrant pain, jaundice, confusion and hypotension

110
Q

Which of the following situations has the least risk of local recurrence and metastasis after removal of a colon polyp while preserving its integrity which contains malignant cells in pathological examination?

Presence of invasive carcinoma in a sessile polyp
Presence of invasive carcinoma in the head of pediculed polyp
Presence of lymphovascular invasion
Larger than 2cm diameter
Histologically poor differentiation

A

Presence of invasive carcinoma in the head of pediculed polyp

NOT: Complete polypectomy in pediculed polyps is a curative intervention if the carcinoma is located at the head of the polyp

111
Q

Which of the following is the most common type of cancer encountered in esophagus?

GIST
Lymphoma
Adenocancer
Leiomyoma
Squamous cell cancer

A

Squamous cell cancer

112
Q

44-y.o. patient brought to the emergency department after a car accident required intubation and mechanical ventilation. The patient’s history was significant for a
genetic deficiency of plasma cholinesterase. A muscle relaxant was administered intravenously. Which of the following muscle relaxants would be contraindicated
in this patient?

Tubocurarine
Cisatracurium
Succinylcholine
Baclofen

A

Succinylcholine

NOT: Succinylcholine is a neuromuscular blockers metabolized by plasma cholinesterase. In patients with a genetic deficiency of normal plasma cholinesterase, these drugs can cause apnea for several hours and are therefore contraindicated.

113
Q

Which one is wrong about capnography used in advanced life support?

*Peak levels are measured at the end of breathing.
*The exhaled O2 levels are monitored in capnography.
*Ineffective chest compressions cause low PETCO2(<10mmHg) values
*Capnography is recommended for tracheal tube location and CPR monitoring
*An important indicator of ROSC is the increase of PETCO2 to around 35-40mmHg.

A

*The exhaled O2 levels are monitored in capnography. This statement is wrong, others are right.

NOT: Capnography used to EXHALED CO2 levels are monitored in the trace.

114
Q

What is the main component of the granulation tissue by mass?

Glycosaminoglycans
Neutrophils
Capillary blood vessels
Collagen
Macrophages

A

Glycosaminoglycans

NOT: Glycosaminoglycans comprise a large portion of the “ground substance” that makes up granulation tissue.

115
Q

Which of the following information is wrong?

*The free end of the appendix can be found in different localizations according to the cecum.
*Appendix is part of the lymphoid system in the gastrointestinal system
*The Appendix specifically produces IgA
*Appendix can be totally absent or can develop as a diverticular process.
*The base of the appendix and its junction with the cecum can be found in different localizations

A

*The base of the appendix and its junction with the cecum can be found in different localizations, this statement is wrong.

NOT: Appendix extends vertically in the newborn, and the location of its free end may change over time, however the localization of the base where the appendix connects to the cecum is fixed and does not change after its development.

116
Q

A 51-year-old patient was an unrestrained driver in a motor vehicle crash in which he sustained multiple traumatic injuries. He is on mechanical ventilation, and has received 8 units of packed red blood cells, 4 units of fresh-frozen plasma, and 6 units of platelets. His arterial blood gas reveals a metabolic alkalosis. The most likely explanation for this finding is:

Under-resuscitation
Hypoventilation
Continued bleeding
Metabolism of citrate to bicarbonate

A

Metabolism of citrate to bicarbonate

NOT: The citrate in the blood preservative is metabolized to bicarbonate by the liver and can cause a metabolic alkalosis following a large volume transfusion. Under-resuscitation and bleeding are likely to cause a metabolic acidosis, whereas hypoventilation causes a respiratory acidosis

117
Q

Mechanisms that may cause hypoxemia under anesthesia include all of the following EXCEPT

Increase in functional residual capacity
Supine position
Hyperventilation
Hypoventilation
Increased airway pressure

A

Increase in functional residual capacity

NOT: (Anesthesia usually causes a decrease in functional residual capacity, which leads to hypoxemia. All of the other options can lead to hypoxemia: hypoventilation by decreased functional residual capacity and increase shunt, hyperventilation by shift of the oxyhemoglobin dissociation curve to the left and decreased cardiac output, supine position by decreased ventilation and decreased functional residual capacity, and increased airway pressure by change in the ventilation perfusion relationships.)

118
Q

During a routine screening mammography, a62-year-old teacher is informed that she has changes on her mammography, and she should consult her physician.
She can be reassured that the findings that indicate a benign condition are which of the following?

*Solid, clearly defined mass with irregular edges
*Fine, clustered calcifications
*Coarse macrocalcifications
*Discrete, nonpalpable mass that has enlarged when compared with a mass shown on a mammogram taken 1 year prior.
*Discrete, stellate mass

A

*Coarse macrocalcifications

NOT: Coarse calcifications are usually benign. Fine, clustered
calcifications are often malignant and require biopsy. Solid tumors of the breast, especially those that have increased in size or have changed in appearance, are suspicious for carcinoma and require biopsy.

119
Q

Which of the following is preferred primarily for evaluating wall invasion in rectal cancer?

Positron Emmission Tomography
Endorectal Ultrasonography
Lower Abdominal Tomography
Angiography

A

Endorectal Ultrasonography

NOT: Endorectal Ultrasound is primarily necessary for evaluating rectal wall invasion and also lymph node status in rectal cancer

120
Q

An 83 years old male patient, with a serious abdominal pain increasing in severity for about 11 hours, was admitted to emergency service. In physical examination; blood pressure is 84/54 mmhg, hearth rate is 119/min, body temperature is 37.5 °C and saturation is 86%. He has a distension in abdominal wall and tenderness. In Complete Blood Count (CBC), leukocyte count is 44000/mm³, creatinine is 2.4
mg/dl, and lactate is 8.1mmol/L. In Computed Tomography Scan; diffuse increased wall thickness of both small and large intestine and intramural air are detected. Which of the following is the most likely diagnosis for this patient?

Sigmoid Diverticulum Perforation
Acute Mesenteric Ischemia
Inflammatory Bowel Disease
Acute Pancreatitis
Intestinal Perforation

A

Acute Mesenteric Ischemia

121
Q

Which is not seen in a neurogenic shock?

Bronchoconstriction
Increased capillary permeability
Stimulation of the Sympathetic Nervous System
Release of the thrombocyte activating factor
Vasodilatation

A

Stimulation of the Sympathetic Nervous System

NOT: In neurogenic shock, the vasomotor centers and the sympathetic system are blocked of due to the CNS being affected.

122
Q

Which of the following benign liver tumors have the potential for rupture and bleeding and might also have a precancerous tendency and hepatocellular cancer may develop within years?

Adenoma
Focal nodular hyperplasia
Cavernous hemangioma
None

A

Adenoma

NOT: Hepatocellular adenomas have the potential for hemorrhage and HCC (hepatocellular cancer) development.

123
Q

Which of the following is not a risk factor for gastric cancer?

Blood type group A
Smoking
Obesity
GERD
Alcohol abuse

A

GERD

NOT: Gerd is a risk for esop ca not stomach.

124
Q

Which of the following tools for pain intensity assessment is a multi-dimensional one?

Binary scale
Mc Gill pain questionnaire
Visual analogue scale
Numerical rating scale
Faces rating scale

A

Mc Gill pain questionnaire

125
Q

Which group of drugs is NOT used for general anesthesia?

Propofol
Non-steroidal analgesics
Rocuronium
Thiopenthal
Isoflurane

A

Non-steroidal analgesics

NOT: Non-steroidal analgesics are used as an adjunct to anesthesia for postoperative analgesia. The other ones are used in induction and maintenance of anesthesia

126
Q

A 68 year old woman admits to the emergency room with left lower quarant pain, fever and vomiting. CT scan reveals a thickened sigmoid colon with inflamed diverticula along with an 8 cm rim-enhancing abscess (Hinchey II). Which of the following is the most apprepriate next step in this patients management?

Sigmoid resection with an anastomosis
Sigmoid resection with end colostomy
Long-term suppressive antibiotic therapy
Total colectomy
Percutaneus drainage of abscess

A

Percutaneus drainage of abscess

NOT: Yaygın peritonit yok, lokal abse nedeniyle perkütan drenaj gerekli

127
Q

Which of the following is classified as primary peritonitis?

Peritonitis due to Systemic Lupus Erythematosus (SLE)
Peritonitis due to Meckel diverticulitis
Peritonitis due to gallbladder perforation after acute stony
cholecystitis
Peritonitis due to peptic ulcer perforation.
Peritonitis due to colon diverticulitis.

A

Peritonitis due to Systemic Lupus Erythematosus (SLE)

BU YANLIŞ MI

128
Q

55-year-old woman was recovering from hysterectomy and started an opioid analgesic for postoperative pain management. Which of the following molecular actions most likely mediated the analgesic effect of the drug?

A

Activation of µ (mu) receptors

129
Q

49-y.o. patient undergoes surgery to repair a small bowel closed loop obstruction. Which of the following general anesthetics would be contraindicated in this patient?

A

Nitrous oxide

130
Q

67-year-old woman underwent hysterectomy for endometrial carcinoma. The general anesthesia protocol included sodium thiopental, isoflurane, nitrous oxide, and tubocurarine. The anesthesiologist also administered another drug to counteract tubocurarine-induced hypotension due to histamine release. To which of the following classes did this drug most likely belong?

A

H1 antagonists

131
Q

57-y.o. patient with complaining of severe pain has been injected with morphine posoperatively. Which of the following adverse effects most likely occurred in this patient?

A

Euphoria

132
Q

What is the rationale of using multimodal analgesia (using medicines with different and complementary mechanisms of action with opioids) for post-operative pain management?

A

Minimizing the requirements for opioids.

133
Q

36-year-old patient had been recovering from surgery and received an intravenous injection of paracetamol to maintain postoperative pain. What is the
mechanism of action of this drug?

A

Decreased concentration of prostaglandins

134
Q

Which of the following drugs would be contraindicated in this patient with dehydration and increased risk of renal impairment after surgery?

A

Ibuprofen

135
Q

44-year-old woman was preparing to undergo surgery to remove a biliary calculus. The woman had suffered from T-cell lymphoma treated successfully with a four-drug combination that included prednisone. The therapy ended 6 months previously. The physician prescribed a course of prednisone treatment before the upcoming surgery.
Which of the following was most likely the primary goal for restarting prednisone treatment in this patient?

A

To decrease nausea and avoid adrenal insufficiency after surgery

136
Q

Which of the following agents cannot be used for post-operative nausea and vomiting?

A

Tramodol

137
Q

39-year-old patient received an intravenous injection of dexamethasone after the surgery. What is the rationale using dexamethasone postoperatively?

A

Treatment of postoperative nausea and vomiting

138
Q

A 45-year-old male is referred to the clinic for a left-sided adrenal mass. Serum metanephrines are markedly elevated and are scheduled for laparoscopic left adrenalectomy. Prior to proceeding to the operating room, what intervention should be performed?

A) No further intervention is required. Proceed with surgery.
B) Left-sided adrenal biopsy to rule out cancer
C) Alpha-blockade and then the addition of beta-blockade 10 days prior to surgery
D) Hydration followed by beta-blockade.
E) Beta-blockade until the patient is not tachycardic and then the addition of an alpha blocker

A

C) Alpha-blockade and then the addition of beta-blockade 10 days prior to surgery

139
Q

What is most likely diagnosis in a 24 years old female patient presenting with a breast lump?

Ductal carcinoma
Fibroadenoma
Lobular carcinoma
Invasive carcinoma
Periductal mastitis

A

Fibroadenoma

140
Q

Which of the following is the first phase of wound healing?

Synthesis
Hemostasis
Remodeling
Inflammation
Epithelization

A

Hemostasis

141
Q

What does “MAC” stand for?

Minimal analgesic concentration
Minimal anaesthetic concentration
Minimum alveolar concrentation
Maximal alveolar concentration

A

Minimum alveolar concrentation

142
Q

An 18 year man is brought to an urban emergency department by emergency medical services after friends found him minimally responsive on the floor of his apartment. On arrival, his airway is patent, his respirations are rapid and deep, and you note his breath has a fruity odor. He is hemodynamically stable but somewhat tachycardia with slight decrease in pulse volume. His initial chemistries are as follows: sodium 134 mEq/L, potassium 5.6 mEq/L, chloride 94 mEq/L, bicarbonate 10 mEq/L, and glucose 890 mg/dL. His initial arterial blood gas measurements while receiving room air as follows: pH: 7.15, PaCO2: 14 mm Hg, PaO2: 87 mmHg, and bicarbonate 10 mEq/L. Which of the following acid-base disturbances are present in this patient?

Normal-gap metabolic acidosis, respiratory alkalosis
High-anion gap metabolic acidosis, respiratory alkalosis
High-anion gap metabolic acidosis, normal-gap metabolic acidosis
High-anion gap metabolic acidosis, metabolic alkalosis, respiratory alkalosis
High-anion gap metabolic acidosis, normal-gap metabolic acidosis, respiratory alkalosis

A

High-anion gap metabolic acidosis, respiratory alkalosis

143
Q

When considering oxygen transport in the lung, Which is the least important cause of hypoxemia?

Altitude
Venous admixture
Ventilation/perfusion mismatch
Bronchial artery blood flow
Diffusion barrier

A

Venous admixture

144
Q

A 45 year old man presents with a perianal pain, discharge (pus) and fever. She has tender, fluctuant, erythematous subcutaneous lump on rectal examination. Which of the following is the most likely diagnosis?

Anal fissure
Pilonidal abscess
Fistula
Perianal abscess
Perirectal abscess

A

Perianal abscess

145
Q

Which of the following is not a risk factor for esophageal cancer?

Diet rich in red meat
Obesity
Gastroesophageal reflux
Corrosive substance burns
Ethanol abuse

A

Diet rich in red meat

146
Q

A 23-year-old woman presents with intermittent right upper epigastric abdominal pain and nausea. E She is afebrile with normal vital signs. On laboratory findings, she has normal levels of bilirubin alkaline phosphatase and no leukocytosis. The amylase and lipase are elevated. Ultrasound demonstrates gallstones, normal gallbladder wall thickness, no pericholecystic fluid, and a common bile duct of 3 mm. What is the most appropriate diagnosis?

Acute cholelithiasis
Cholangitis
Choledocholithiasis
Symptomatic cholelithiasis
Gallstone pancreatitis

A

Gallstone pancreatitis

147
Q

Hyper-dynamic shock is frequently also referred to as distributive or vasodilatory shock. The following shock causes are examples of this type of shock, except

Sepsis
Anaphylaxis
Neurogenic shock
Myocardial infarction induced shock
Adrenal crisis

A

Myocardial infarction induced shock

148
Q

Which of the following is the most common cause of an acute hemolytic transfusion reaction?

An error during type and screen
Misidentification of the patient, blood specimen, or transfusion unit
An error during type and crossmatch
Defective blood filter

A

Misidentification of the patient, blood specimen, or transfusion unit

149
Q

In the clinical diagnosis of brain death, which of the following is not required?

Irreversible coma with known cause
Fulfillment of prerequisites
Absence of brainstem reflexes
Apnea
Cessation of the body circulation

A

Cessation of the body circulation

150
Q

Which of the following is not a precondition of the apnea test for the diagnosis of brain death?

The patient should be well oxygenated with PEEP 5 cm H2O.
The patient’s hemodynamics must be stable
Ventilator settings should be made as normocarbi (PaCO2 35-45 mmHg)
The patient should have spontaneous breathing
The patient should be inhaled with 100% FiO2 for 10 minutes until PaO2> 200 mmHg.

A

The patient should have spontaneous breathing

151
Q

Which one defines the common bile duct (CBD)?

The common hepatic duct and pancreatic duct merge to form the CBD
The right and left hepatic ducts merge to form the CBD
The right hepatic duct and cystic duct merge to form the CBD
The common hepatic duct and cystic duct merge to form the CBD

A

The common hepatic duct and cystic duct merge to form the CBD

152
Q

All are amides except …

Prilocaine
Procaine
Etidocaine
Lidocaine
Bupivacaine

A

Prilocaine

153
Q

Which of the following is preferred primarily for evaluating wall invasion in rectal cancer?

Virtual Colonoscopy with Computed Tomography
Endorectal Ultrasonography
Angiography
Lower Abdominal Tomography
Positron Emmission Tomography

A

Endorectal Ultrasonography

154
Q

A 55-year-old man presents with intermittent right upper quadrant abdominal pain. Each episode of pain lasts1 to 2 hours. He is afebrile with normal vital signs. On laboratory findings, he has normal levels of bilirubin amylase,
alkaline phosphatase, and no leukocytosis. Ultrasound demonstrates gallstones, normal gallbladder wall thickness, no pericholecystic fluid, and a common bile duct of 3 mm. What is the most appropriate diagnosis?

Choledocholithiasis
Gallstone pancreatitis
Cholangitis
Acute cholelithiasis
Symptomatic cholelithiasis

A

Symptomatic cholelithiasis

155
Q

Which one is not among the palliative surgery indications in the treatment of gastric cancer?

obstruction
perforation
bleeding
weight loss
all the above

A

all the above

156
Q

The anatomical right and left hemilivers are separated by an imaginary line which is also known as the Cantlie line running between:

Segment 1 and segment 8
Gallbladder fossa and inferior vena cava
The falciform and coronary ligaments
Caudate lobe and portal vein

A

Gallbladder fossa and inferior vena cava

157
Q

In general, herbal medications should be stopped before surgery for at least
______ days.
Which of the following fills in the blank correctly?

1
3
7
10
14

A

7

158
Q

A 43-y.o. man underwent an emergency minor arm repair procedure after a car accident. A standard dose of lidocaine was administered near the brachial plexus for peripheral nerve block. Fifteen minutes later, the anesthesia was
still incomplete, and another dose of lidocaine was administered. Which of the following adverse effects would most likely occur after the administration?

Ventricular tachycardia
Abdominal colic
Drowsiness
Hypertensive crisis

A

Ventricular tachycardia

159
Q

Which of the following is not among the etiologies of secondary peritonitis?

Bile duct injury during the procedure in a patient with cholangitis.
Diffuse acid-related peritonitis in a patient with liver cirrhosis.
Gallbladder perforation due to cholecystitis.
Colon perforation.
Acute necrotizing pancreatitis.

A

Acute necrotizing pancreatitis.

160
Q

Which of the following is wrong?

Peritonium suspend the organs within the peritoneal cavity.
Peritonium is the largest serous membrane in the body.
Peritonium, divided in to parietal and visceral portions.
Peritonium secretes the peritoneal fluid.
Parietal layer, covers the abdominal and pelvic viscera and includes the mesenteries

A

Peritonium is the largest serous membrane in the body.

(NE PEKİ)

161
Q

Which of the following situations is a risk factor for cholangitis development?

Periampullary cancer
Choledochal cysts
Choledocholithiazis
All of the above

A

All of the above

162
Q

Which of the following is true for the esophagus?

Food is delivered to stomach with gravity
Its lumen is covered with cylindrical epithelium
The outer surface is covered by a thick layer of serosa
It is a muscular tube and its lumen is covered by squamous epithelium
It consists of 4 seperate anatomical parts

A

It is a muscular tube and its lumen is covered by squamous epithelium

163
Q

A patient is experiencing unilateral vocal cord paralysis post-operatively after thyroidectomy for follicular thyroid cancer. What is the most likely reason for this?

Recurrent laryngeal nerve damage during surgery
Superior laryngeal nerve damage during surgery
11th cranial nerve injury during surgery
Radioactive iodine treatment side effect

A

Recurrent laryngeal nerve damage during surgery

164
Q

Which of the following clinical scenarios is most suspicious for thyroid cancer?

1) A 22 year old woman with enlarged thyroid gland and tenderness on exam

2) A 27 year old asymptomatic male with a hard fixed 2 cm nodule on examination

3) A 33 year old woman with a history of hypothyroidism with a 1 cm mobile nodule on examination

4) A 30 year old man with symptoms of excessive sweating, weight loss, and hyperactivity and with exophthalmos and a slightly enlarged thyroid gland on exam without any palpable nodules

5) A 36 year old woman with a history of hypothyroidism on levothyroxine treatment for the past 8 years with a slightly enlarged boggy feeling thyroid gland on examination without any distinct nodules

A

2) A 27 year old asymptomatic male with a hard fixed 2 cm nodule on examination

165
Q

A patient with cerebral edema would most likely be ordered what type of solution?

% 5 dextroz
Ringer lactate solution
% 0,45 sodium cloride
%3 saline (hypertonic)
Izotonic sodium cloride

A

%3 saline (hypertonic)

166
Q

Which of the following is not the physiological mechanism responsible for anaphylactic shock?

Reduced cardiac output
Vasodilatation
Increased capillary permeability
Increased ventricular filling
Hypoxemia

A

Increased ventricular filling

167
Q

Which of the following clinical parameters is not a marker for adequate fluid resuscitation in a patient with hypovolemic shock?

Average blood pressure > 60 mmHg
Urine output < 0.5 ml/kg/h
Heartrate<120BPM

A

Urine output < 0.5 ml/kg/h

168
Q

Which of the following methods are both diagnostic and therapeutic in a patient with suspected mesenteric artery embolism or thrombosis?

Angiography
Plain X-Ray
USG
CT Angiography
MRI Angiography

A

Angiography

169
Q

Which of the following is a risk factor for developing HCC (hepatocellular cancer)?

Chronic HBV infection
Hemochromatosis
Chronic alcohol abuse
All of the above

A

All of the above

170
Q

Which of the following is not a direct risk factor for pancreatic cancer?

A diet rich in red meat
Smoking
Alcohol abuse
Male sex
Obesity

A

Alcohol abuse

NOT: There is no clear correlation between alcohol consumption and pancreatic cancer. However, ethanol abuse can cause acute or chronic pancreatitis.

171
Q

Which of the following statements relating to blood volume is NOT true?

Iv fluids increase preload. .

Administration of isotonic İV fluids increases circulating blood volume.

An increase in stroke volume will cause an increase in cardiac output .

An increase in cardiac contractility leads to an increase in preload.

Stroke volume increases as pre-load increases upto some point.

A

An increase in cardiac contractility leads to an increase in preload.

172
Q

A 70-year-old man presents with epigastric pain and iron deficiency anemia. Upper endoscopy reveals a large, ulcerated mass in the gastric antrum. Biopsy confirms the diagnosis of gastric adenocarcinoma. Further imaging reveals regional lymph node involvement and possible peritoneal insemination. What is the most appropriate next step in the management of this patient?

Initiate targeted therapy with trastuzumab and chemotherapy based on molecular testing results

Administer palliative chemotherapy to control symptoms and improve quality of life

Offer supportive care only, as the patient’s advanced age and disease extent preclude aggressive treatment

Perform surgical re-staging of the tumor and resect if appropriate

A

Perform surgical re-staging of the tumor and resect if appropriate

173
Q

Following surgical resection of a large thyroid mass, a patient complains of persistent hoarseness and a weak voice. What is the most likely cause of these symptoms?

Traumatic intubation
Scar tissue extending to the vocal cords
Injury to the recurrent laryngeal nerve
Injury to the superior laryngeal nerve

A

Injury to the recurrent laryngeal nerve

174
Q

A 36-year-old woman presents with palpitations, anxiety, and hypertension. Workup reveals a pheochromocytoma. Which of the following is the best approach to optimizing the patient preoperatively?

Initiation of an α-blocker 1 hour prior to surgery
Fluid restriction 24 hours preoperatively to prevent intraoperative congestive heart failure
Initiation of an α-blocker at 1 to 3 weeks prior to surgery
Initiation of a β-blocker 1 to 3 weeks prior to surgery

A

Initiation of an α-blocker at 1 to 3 weeks prior to surgery

175
Q

Which of the following structures is not involved in the nerve conduction of pain?

Thalamus
Hypothalamus
Anterior horn cells
Brainstem
Dorsal root ganglia

A

Anterior horn cells

176
Q

Which of the following will cause a shift of potassium ions from the intracellular to the extracellular compartment?

Thyrotoxicosis
Metabolic acidosis
Hyposmolarity
Ingesting sodium
Respiratory alkalosis

A

Metabolic acidosis

Acidosis: Acidosis, characterized by an accumulation of acid
in the body, can cause potassium ions to shift from the
intracellular compartment to the extracellular compartment. In
acidosis, the intracellular pH decreases, leading to an
increase in the intracellular concentration of hydrogen ions. To
maintain electroneutrality, cells shift potassium ions out of the
cells and into the extracellular fluid.
Insulin deficiency: Insulin stimulates the uptake of glucose and
potassium by cells. In conditions such as diabetic
ketoacidosis, where there is a deficiency of insulin, cells
cannot effectively take up glucose and potassium, leading to
hyperglycemia and hyperkalemia.
Cell lysis: In conditions such as rhabdomyolysis, tumor lysis
syndrome, or hemolysis, the lysis of cells releases large
amounts of potassium into the extracellular fluid, leading to
hyperkalemia.
Medications: Certain medications can cause a shift of
potassium ions from the intracellular to the extracellular
compartment. Examples include beta-blockers, which can
inhibit the Na+/K+ ATPase pump, leading to a decrease in
intracellular potassium concentration

177
Q

Which of the following is not a pancreatic digestive enzyme that leaks into the systemic circulation and is elevated in serum?

Alkaline phosphatase
Carboxyl ester lipase
Trypsin
Phospholipase

A

Alkaline phosphatase

178
Q

Which of the following is not correct about rescue breaths applied during CPR on an adult patient?

Tidal volume should be given enough to raise the chest
Before starting chest compressions 2 rescue breaths should be given
Each should take longer than 1 second
Lips should cover the mouth
Give the second breath as soon as the chest wall descends

A

Before starting chest compressions 2 rescue breaths should be given

179
Q

Which of the following is a key imaging modality for the diagnosis of acute cholecystitis?

Cholescyntigraphy
Computed tomography (CT) scan
Magnetic resonance imaging (MRI)
Ultrasonography (US)

A

Ultrasonography (US)

180
Q

Which of the following causes organ damage from shock?

Combination of the ischemic injury and inflammatory mediators.
Combination of the ischemic injury, reperfusion injury, and
inflammatory mediators
Ischemic injury
Reperfusion injury
Inflammatory mediators

A

Combination of the ischemic injury, reperfusion injury, and
inflammatory mediators

181
Q

Which of the following is the most common cause of acute appendicitis in the elderly population?

Vascular circulation disorder
Constipation
Lymphoid hyperplasia
Ascariasis

A

Vascular circulation disorder

182
Q

Which of the following decreases potassium excretion?

Decreased urine flow rate
Decreased sodium plasma concentration
Increased Na/K ATPase activity
Increased distal sodium delivery
Increased aldosterone release

A

Decreased urine flow rate

183
Q

Which of the following is the most common and most typical symptom in chronic mesenteric ischemia?

Diarrhea
Postprandial abdominal pain
Nausea
Vomiting

A

Postprandial abdominal pain

184
Q

Trauma patients should literally be examined from head to toe to prevent missed injuries. Which of the following body areas is not among the commonly overlooked sites?

Thorax
Axilla
Perineum
Scalp

A

Thorax

185
Q

A 35-year-old man presents to his primary care physician for a routine check-up. He reports a family history of colorectal cancer in his mother, sister, and two maternal aunts. All of these relatives were diagnosed at an early age (before age 50). The patient is concerned about his own risk of developing colorectal cancer and wants to know if genetic testing is appropriate. Which of the following statements is true?

A genetic study of this patient might reveal autosomal dominant mutations in DNA mismatch repair genes.
A genetic study of this patient might reveal mutations in the APC gene.
His condition is only associated with an increased risk of colorectal cancer.
A genetic study of this patient might reveal mutations in the BRAF gene.

A

A genetic study of this patient might reveal autosomal dominant mutations in DNA mismatch repair genes.

186
Q

Which of the following situations is not a risk factor for cholangitis development?

Choledocholithiazis
Periampullary cancer
Choledochal cysts
Cholecystectomy

A

Cholecystectomy

187
Q

Etiologies associated with hypocalcemia may include all of the following except:

Vitamin D deficiency
Inadequate intake of calcium
Metastatic bone lesions
Renal failure

A

Metastatic bone lesions

188
Q

Which of the following is not a reason why serum lactate is checked in sepsis patients?

It has a high specificity for sepsis.
Lactate is a by-product of anaerobic metabolism.
It may be the earliest clue in well-compensated patients.
It has a very good sensitivity for sepsis.
It is a surrogate marker for the adequacy of tissue-level
perfusion.

A

It has a high specificity for sepsis.

189
Q

Which of the following is not a risk factor for symptomatic hemorrhoids?

Chronic constipation
Diarrhea
Straining
Younger age

A

Younger age

190
Q

Which of the following is the most common cause of mechanical small bowel obstruction?

Tumors
Adhesions
Crohn’s disease
Complicated hernias

A

Adhesions

191
Q

Which of the following changes is not observed in chronic pancreatitis?

Pancreatic duct obstruction
Periglandular and parenchymal fibrosis
Hypertrophy of the gland due to chronic injury
Parenchymal calcifications

A

Hypertrophy of the gland due to chronic injury

192
Q

Which of the following is part of the legal criteria for the neurological determination of death?

Established etiology capable of causing cardiac death
Clinical evidence of reversible causes of coma
Absent brainstem reflexes
Observable respiratory effort during the apnea test

A

Absent brainstem reflexes

NOT: The neurological determination of death (NDD) is a legal and
medical definition used to declare death in individuals who have
suffered a brain injury. The criteria for NDD varies depending on the
country or jurisdiction, but typically include the following:
Irreverreversible cessation of all brain function
Exclusion of confounding factors
The passage of an appropriate observation period

193
Q

Which of the following is not encountered in the history of acute cholecystitis?

*Patients frequently will have a positive Murphy’s sign
*The pain may radiate to the right shoulder or back
*There is often a history of fatty food ingestion one hour or more before the initial onset of pain
*Patients with acute cholecystitis typically complain of abdominal pain, most commonly in the right upper quadrant or epigastrium

A

*Patients frequently will have a positive Murphy’s sign

NOT: Patients frequently will have a positive Murphy’s sign. The Murphy’s sign is a physical examination finding, not a component of the patient’s history

194
Q

What type of antibody is produced during initial antigen exposure in a Type I hypersensitivity reaction?

IgA
IgM
IgG
IgD
IgE

A

IgE

NOT: During the initial antigen exposure in a Type I hypersensitivity reaction, the immune system produces immunoglobulin E (IgE) antibodies. IgE is a type of antibody that is normally produced in response to parasitic infections, but in people with allergies, it can also be produced in response to harmless environmental allergens, such as pollen, dust mites, and animal dander. When a person with an allergy is exposed to an

195
Q

Which one is a side effect of thiopental sodium?

Analgesia
Malignant hyperthermia
Dissociative anesthesia
Rapid onset induction
Hypotension

A

Hypotension

196
Q

Which of the following is not among the four potentially life threatening sites of trauma that should be checked in the primary survey of the patient?

Thorax
Pericardium
Pelvis
Retroperitoneum

A

Retroperitoneum

197
Q

A 52-year-old undergoes a left modified radical mastectomy for 2-cm breast cancer. She should be informed that the factor which has the most significant impact on her prognosis is?

Positive findings on tests for the presence of the BRCA (breast cancer)1 gene
The histological type of the carcinoma
The number of axillary nodes positive for metastasis
Hormonal receptor status of the primary tumor

A

The number of axillary nodes positive for metastasis

198
Q

A 65-year-old male, active smoker construction worker who was investigated for chronic fatigue was diagnosed with iron deficiency anemia. He was further investigated with upper and lower gastrointestinal endoscopy and a large tumor was observed in the cecum. The biopsies revealed a mucin rich adenocarcinoma of the colon. Which of the following is more likely regarding the characteristics of this patient’s tumor?

The tumor may cause obstruction early in the course of the
disease.
The tumor may have a KRAS mutation.
The tumor may have a BRAF mutation
The tumor has a favorable prognosis.

A

The tumor may have a BRAF mutation

NOT: BRAF mutations are most commonly observed in sporadic right sided tumors in patients with a history of smoking. These tumors tend to have a poor prognosis.
Although occasional KRAS mutations may be present, these types of mutations are observed in only 15% of sporadic cases. Lastly, the tumors in the cecum rarely cause an obstruction as this part of the colon has the most reserve to get dilated, in fact, this is particularly the reason that right sided colon tumors are mostly diagnosed with unexplained anemia in otherwise healthy individuals.

199
Q

There are several genetic pathways that may ultimately lead to colorectal cancer. Which of the following is generally accepted as a genetic hallmark of hereditary nonpolyposis colorectal cancer syndrome?

Mutations in the BRAF gene
Loss of heterozygosity in the chromosome
Methylation of CpG islands of the DNA
Mutations in DNA mismatch repair genes

A

Mutations in DNA mismatch repair genes

200
Q

Which of the following is the most common presentation of appendix carcinoma?

Mechanical obstruction
Acute appendicitis
Gastrointestinal bleeding
Abdominal ascites

A

Acute appendicitis

201
Q

Which of the following is false for an inguinal hernia physical examination?

*The presence of a dilated external inguinal canal would be found
*The finger should be introduced through the external inguinal ring into the inguinal canal
*The patient should be supine in a relaxed position
*The patient is requested to cough and the indirect hernia should strike the fingertip

A

*The patient should be supine in a relaxed position

NOT: Ideally, a patient should be examined in a standing position for gravity to help hernia findings become obvious. Moreover, patients should be asked to do a Valsalva maneuver.

202
Q

Which of the following is not the initial treatment for acute pancreatitis?

Pain control
Antibiotics
Fluid replacement
Emergency surgical treatment

A

Emergency surgical treatment

NOT: Emergency surgery is not always necessary or indicated in cases of acute pancreatitis. In fact, the vast majority of cases of acute pancreatitis are treated without surgery. The primary treatment for acute pancreatitis is usually supportive care, such as intravenous fluids, pain management, and nutritional support.

203
Q

A 45-year-old male presents to your clinic with a 2-month history of abdominal discomfort and fatigue. Physical examination reveals a palpable, non-tender mass in the right upper quadrant of the abdomen. Laboratory studies are notable for mild elevations in liver function tests. Imaging studies reveal a 5 cm hepatic mass. What is the most appropriate next step in the management of this patient’s liver mass?

Schedule the patient for a biopsy of the liver mass
Initiate treatment with antiviral therapy for hepatitis B or C
Perform imaging studies for further characterization of the liver mass
Refer the patient for surgical resection of the liver mass

A

Perform imaging studies for further characterization of the liver mass

NOT: Before moving on with any definitive therapeutic procedures and/or invasive diagnostic studies; all liver masses should be adequately evaluated with advanced imaging techniques such as MRI or CT as these imaging modalities have the potential to establish a differential
diagnosis and also provide insights about the probable primary cause of a secondary liver mass.

204
Q

A 45-year-old woman presents to her physician with complaints of constipation and bloating. She is an active smoker and, has a medical history significant of IBD with chronic diarrhea as opposed to her current complaints. A colonoscopy is performed, which reveals a large mass in the
transverse colon. Biopsy confirms the diagnosis of adenocarcinoma. Which of the following is a risk factor for the development of colon cancer in patients with IBD?

Use of non-steroidal anti-inflammatory drugs (NSAIDs)
Chronic diarrhea
Age younger than 50 years
Smoking

A

Chronic diarrhea

205
Q

Which one of the following is not part of the management of a patient with hyperparathyroidism?

Vitamin D
Parathyroid scan
Steroids
Hydration with intravenous normal saline

A

Steroids

NOT: The definitive treatment of hyperparathyroidism is
parathyroidectomy. When serum calcium is above 14 mg/dL the patient is in a hypercalcemic crisis. Immediate treatment of this condition requires hydration with normal saline and diuretics to reduce the serum calcium level. A positive parathyroid scan will help locate the parathyroid’s adenoma preoperatively.

206
Q

Which of the following is false for Bochdalek hernia?

Bochdalek hernia is the most common presentation of congenital diaphragmatic hernias accounting for more than 95% of the cases.
The majority of Bochdalek hernias (80-85%) occur on the right side of the diaphragm
Cause is unknown
It can be defined as a posterolateral diaphragmatic hernia

A

The majority of Bochdalek hernias (80-85%) occur on the right side of the diaphragm
This statement is false

207
Q

Which of the following structures is not involved in the nerve conduction of pain?

Brainstem
Dorsal root ganglia
Thalamus
Hypothalamus
Anterior horn cells

A

Anterior horn cells

208
Q

Which of the following statements regarding anesthesia is Incorrect?

Induction drugs, such as thiopentone and propofol, have profound effects in reducing cardiac contractility.

The use of opioids will lead to a slowing of the heart rate but have little effect on contractility.

The decrease in peripheral resistance caused by modem anesthetic vapors may allow the heart to contract more fully.

Moder anesthetic vapors will reduce the contractility of the heart) reducing cardiac output

A

Modern anesthetic vapors will reduce the contractility of the heart) reducing cardiac output

209
Q

Which of the following Indications given below for the surgical treatment of hiatal hernias is not true?

Chest pain
Extra-esophageal manifestations
Complicated gastroesophageal reflux
Successful medical management
Patient decision

A

Successful medical management

210
Q

A patient presents with a painful lump near the anal opening, which is tender and may be accompanied by localized swalling and redness. The most likely diagnosis is:

perianal abscess
Hemorrhoid
Anal fistula
Anal fissure

A

perianal abscess

211
Q

Which of the following is an adverse effect of sodium polystyrene sulfonate?

Hypoglycemia
Renal impairment
Hyponatremia
Congenital malformations
Intestinal necrosis

A

Intestinal necrosis

212
Q

What is the IV/IO dose of adrenaline in advanced life support?

1 mg every 3-5 minutes
3 mg every 15 minutes
First dose 150 mg bolus, second dose 300 mg.
First dose 1-1,5 mg/kg , second dose 0,5-0,75 mg/kg
First dose 300 mg bolus, second dose 150 mg

A

1 mg every 3-5 minutes

213
Q

How should the patient be ventilated after providing an advanced airway?

Ventilate the patient at a rate of 2 breathes per minute
Ventilate the patient at a rate of 8-10 breathes per minute
Ventilate the patient at a rate of 20-25 breathes per minute It is no longer necessary to ventilate the patient
Contnue without any changes. (30:2)

A

Ventilate the patient at a rate of 8-10 breathes per minute

214
Q

Which of the following benign liver tumors have the potential for rupture and bleeding and might also have a precancerous tendency and hepatocellular cancer may develop within years?

Adenoma
Cavemous hemangioma
Focal nodular hyperplasia .
None of the above

A

Adenoma

Not: Hepatocellular adenomas have the potential for hemorrhage and HCC (hepatocellular cancer) development

215
Q

which of the following conditions is most commonly mistaken with appendicitis in adults?

Acute mesenteric lymphadenitis
Acute gastroenteritis
Pelvic inflammatory disease
Meckel’s diverticulitis
Rupture of an ovarian cyst

A

Acute mesenteric lymphadenitis

216
Q

Which of the following approaches should not be preferred in the follow-up and treatment of incidental adrenal masses?

Hormonal evaluation regardless of size
If there is suspicion of malignancy on CT in adrenal masses between 3-5 cm, adrenalectomy
Adrenalectomy for tumors larger than 5 cm Follow-up with CT in nonfunctional tumors smaller than 3 cm
CT-guided fine-needle aspiration biopsy in all adrenal masses between 3-5 cm Response

A

CT-guided fine-needle aspiration biopsy in all adrenal masses between 3-5 cm Response

Feedback: Fine needle aspiration biopsy is not a routine practice

217
Q

in which of the following thyroid diseases, surgery is indicated more frequently in primary treatment because of the nsk of malignancy?

Lymphocytic thyroid
Solitary hypoactive nodule
Hashimoto Thyroiditis
Graves disease
Simple diffuse goiter

A

Hypoactive and solitary nodule’s risk of malignancy is higher than others.

218
Q

50-year-old patient had been recovering from surgery and received paracetamol to maintain postoperative pain. Which of the following molecular actions mediate the analgesic effect of this drug?

Drug binding to prostaglandin receptors
Decreased oxygen radical production
Decreased concentration of prostaglandins
Lowering of anxiety induced by pain

A

Decreased concentration of prostaglandins

219
Q

Gold standart test for peritonitis diagnosis?

Mrı
CT
Parasynthesis
Usg

A

Parasynthesis

220
Q

A previously healthy 27-year-old man comes to the emergency department because of perianal pein, swelling and dramege of a small amount of pus for the pest 2 weeks He reports a 4.5-kg weight loss, abdominal cramping, and intermittent mucus in his stools. He reports heaving 7 to & stools per day. His only medication is loperamide. On physical examination, tis temperature is 37.2°C, his pulse is 88/min and regular, respirations are 14/min, and blood pressure is 120/80 mmHg. The abdomen is flat and soft with mild tendemess to palpation over the right lower quadrant. Rectal examination shows a chronic appearing fistula-in-ano. Which of the following is the most likely diagnosis?

Ulcerative colitis
Colorectal cancer
Chrohn’s disease
Scc of anus
Irritable bowel disease

A

Crohn’s disease

Not: This patient has a classic history of Crohn’s disease, one of the chronic inflammatory bowel diseases. Chronic diarrhea, cramping, and mucus in the stool is a finding often associated with either Crohn’s disease or ulcerative colitis. Ulcerative colitis (UC ), is characterized by mucosa-based inflammation and always involves the rectum with varying degrees of proximal extension in the colon. Perianal disease and fistulas are usually not associated with UC.

Crohn’s disease is characterized by full-thickness inflammation in the Gl tract. There can be areas of normal tissue between areas of inflammation (skip disease) and rectal sparing is common. Perianal disease, (including abscesses, fissures, and fistulae) is common with Crohn’s disease, and some patients develop severe disease with multiple fistulae (watering-can anus).

Patients with colorectal cancer often are asymptomatic. Gross rectal bleeding without diarrhea can be a sign of distal colorectal cancer. Obstructive symptoms are more commortand can be associated with paradoxical diarrhea, but this is usually less severe and not associated with significant amounts of mucous. Perianal disease is usually not associated with colorectal cancer. Patients with colorectal cancer are usually older unless there is a genetic risk such as with familial polyposis.

Perianal squamous cell cancers can occur in young individuals, particularly those with a history of HCV infections. The other symptoms manifested in this patient are rarely present.

Diarrhea can be a prominent symptom in some patients with irritable bowel syndrome (IBS), but the perianal disease is not a manifestation of IBS.

221
Q

A 19-year-old man presents to the emergency department with progressive right lower quadrant abdominal pain over the last 24 hours, beginning while helping a friend move heavy fumiture. He reports nausea, anorexia, and fever to 38.2°C at home. He has not had a bowel movement since the symptoms began.

Which of the following components of the physical exam will be most useful in establishing the diagnosis?

Asking the patient to sharply inhale while palpating under the liver edge

Digital rectal examination

Palpation of the inguinal canal through the scrotum

Inspection of the abdomen

Auscultation of the abdomen

A

Palpation of the inguinal canal through the scrotum

It will be important to evaluate the patient for an incarcerated and strangulated inguinal hernia as an etiology for his fever and right Feedback: lower quadrant abdominal pain, which can most effectively be performed by palpation of the inguinal canal through the scrotum. Observation may also demonstrate a bulge or erythematous skin changes in this region. Auscultation of the abdomen is unlikely to provide more information than direct palpation. Asking the patient to inhale sharply while palpating under the inferior edge of the liver to elicit Murphy’s sign would be more important if the patient complained of right upper quadrant abdominal pain, or had symptoms of cholecystitis. Digital rectal examination may be helpful to evaluate for frank or occult blood, or tenderness, but is unlikely to provide more useful information than evaluation of the inguinal canal in this patient.

222
Q

A 38-year-old woman comes to your office because of a 4-week history of painful bowel movements associated with a small amount of blood on the toilet tissue with wiping. She has no history of similar episodes but is very concerned because her grandfather died of colon cancer. She has occasional constipation but has otherwise been healthy. The patient is in a monogamous relationship with her husband. With the exception of topical hemorrhoid ointment, she takes no medications.

On physical examination, her temperature is 37.0°C, her pulse is 80/min and regular, respirations are 14/min, and her blood pressure is 110/70 mmHg. Physical examination shows a posterior midline skin tag that is tender to palpation. Digital rectal examination cannot be completed because of severe pain and sphincter spasm.

Which of the following is the most likely cause of this patient’s symptoms?

Chronic anal fissure
Perianal abscess
Condyloma accuminata
Thrombosed hemorrhoid
Scc of anus

A

Chronic anal fissure

223
Q

A 50 ylo man is the unrestrained driver in a motor vehicle crash and is admitted to the ED hemodynamically stable. Vital signs are BP 110/70 mmHg. HR 100 bpm. On CT of the abdomen, he is found to have a Grade III splenic laceration. He is admitted to the ICU for close monitoring Six hours after admission he is hypotensive with a blood pressure of 80/40 mmHg and a heart rate of 130 bpm. After transfusion of four units of packed red blood cells, he remains in shock.

What is the most appropriate next step in management?

Diagnostic peritoneal lavage
Emergent laparotomy
Visceral angiography
Continued transfusion
Repeat CT scan

A

Emergent laparotomy

Not: In the past 2 decades, nonoperative management has become the mainstay of management for solid organ injuries in Feedback: hemodynamically stable patients. However, occasionally patients fail nonoperative management and require emergent laparotomy in order to control the ongoing hemorrhage. Grade III splenic injuries are at increased risk for failure of nonoperative management represented by the need for transfusion of four or more units of blood or frank hypotension due to hemorrhagic shock.

224
Q

Which of the following mediciries could be used for prophylaxis against postoperative nausea and vomiting?

Droperidol
Cetirizine
Dexamethazone
Ondanstron

A

Cetirizine

225
Q

A 45-year-old male presents to his family doctor with unexplained weight loss and fatigue. Colonoscopy demonstrates a 2 cm ascending colon mass, from which the biopsies are consistent with adenocarcinoma. A staging CT scan is performed which shows > 10 bilateral hepatic lesions consistent with metastases. CEA is 212 (normal: <3 ng/mL).

Which of the following is the most appropriate course of management?

Laparoscopic right hemicolectomy followed by adjuvant chemotherapy
Staged hepatic resection with radiofrequency ablation (RFA) and subsequent right colectomy
Simultaneous right hemicolectomy with implantation of a hepatic arterial infusion pump (HAI).
Chemotherapy with radiation
Chemotherapy alone

A

Chemotherapy alone

226
Q

Gallbladder contraction is stimulated by the production of a hormone due to the presence of fatty substances in the duodenum. This hormone can be injected directly to assess for functional gallbladder abnormalities that may be present in the absence of gallstones. Please select this hormone.

Cholecystokinin
Gastrin
Leptin
Ghrelin
Secretin

A

Cholecystokinin

227
Q

Kurtarıcı, baygın bir yetişkinle karşılaştığında ilk olarak ne yapmalıdır?

Yardım çağrısı yapma
Entübe etme
AED (otomatik harici defibrilatör) edinme
Olay yeri güvenliğini doğrulama
Hemen CPR’ye başlama

A

Olay yeri güvenliğini doğrulama

228
Q

A 72-year-old woman comes to the emergency department with a 2-day history of vomiting and intermittent colicky abdominal pain. She has not had any recent weight loss. For the past 10 years, she has had postprandial upper abdominal pain that resolves with self-medication with an analgesic. She has no other significant medical history, and she takes no other medications.

On physical examination, her temperature is 96.8°F (36.0°C), her pulse is 90/min and regular, respirations are 22/min, and her blood pressure is 130/90 mmHg. No murmurs are heard. The lungs are clear to auscultation and percussion. The abdomen is distended and mildly tender to deep palpation. Bowel sounds are high-pitched and tinkling. A plain radiograph of the abdomen shows branching air collections projecting over the liver with distended small bowel and decompressed colon.

Which of the following is the most likely diagnosis?

Adhesive small bowel obstruction
Gallstone ileus
Lymphoma
Internal hernia
Diverticulitis

A

Gallstone ileus

229
Q

Which of the following is an absolute Indication for surgical treatment in asymptomatic primary hyperparathyroidism?

If the patient is more than 50 years old

0.5 mg/dl. above normal serum calcium level

Having a glomerular filtration rate below 70 mL/min

20% decrease in creatinine clearance compared to normal

Reduction of bone mineral density by more than 2 standard deviations in both areas

A

Reduction of bone mineral density by more than 2 standard deviations in both areas

Not: 1. Serum calcium > 1 mg/dl. above the upper limit of normal

  1. Bone density T score <-2.5
  2. Vertebral fracture by imaging
  3. Creatinine clearance 60 mL/min
  4. 24-hour urine calcium 400 mg/d and increased stone risk by biochemical stone risk analysis
  5. Presence of kidney stones
  6. Age 50
230
Q

A good local anesthetic agent should cause:

Vasodilatation
Local irritation and tissue damage
Fast onset and long duration of action
Sistemic toxicity
Emesis

A

Fast onset and long duration of action

231
Q

The presence of which cell type from a hypothetical biopsy, proves the transition of wound healing to the proliferation phase?

Platelets
Monocytes
T lymphocytes
Fibroblasts

A

T lymphocytes

232
Q

Which of the following is the blood glucose goal for the treatment of hyperglicemic patient who is in septic shock?

<110 mg/dL
<80 mg/dL
<200 mg/dL
>200 mg/dL
<180 mg/dL

A

<180 mg/dL

Not: If a patient has hyperglycemia and has insulin drip treatment a blood glucose goal is <180mg/dl because hyperglycemia effects immun system and healing

233
Q

Which of the following is not found in the pain triangle of the inguinal region?

Femoral branch of nervus genitofemoralis
Nervus femoralis
Deep circumflex branch of iliac artery and vein
Ductus deferens
Gonadal veins

A

Ductus deferens

234
Q

A 39 year old previously healthy man is hospitalized for 10 days with epigastric pain radiating to his back, nausea and vomiting. Initial laborotory values revealed an elevated amylase level consistent with acute pancreatitis. Six weeks after discharge, he complains of early satiety, epigastric pain and fevers. On presentation, his temperature is 38.9 Celcius degree and his heart rate is 120 beats per minute, WBC count is 24,000/mm3 and his amylase level is normal. He undergoes a CT scan demonstrating a 6 cm by 6 cm rim-enhancing fluid collection in the body of the pancreas.

Which of the following would be the most definitive management of the fluid collection?

Surgical internal drainage of the fluid collection with a cyst-gastrostomy

CT guided aspiration with repeat imaging in 2 to 3 days

Antibiotics and percutaneus catheter drainage

Antibiotic therapy alone

A

Antibiotics and percutaneus catheter drainage

235
Q

A twelve year old girl developes itchy erthythematous skin lesions on her body following consumption of seafood. Which of the following immumological mechanisms would contribute to the development of this case?

C1 esterase deficiency
IgA deficiency
Type 1 hypersensitivity
Immune complex deposition
Delayed type hypersensitivity

A

Type 1 hypersensitivity

236
Q

A 68 year-old man with a history of very severe COPD (FEV1-1.0L, <25% predicted) and chronic carbon dioxide retention (Baseline PCO2 58) presents to the emergency room complaining of worsening dyspnea and an increase in the frequency and purulence of his sputum production over the past 2 days. His oxygen saturation is 78% on room air. Before he is place on supplemental oxygen, a room air arterial blood gas is drawn and reveals: pH 7.25, PCO2 68, PO2 48, HCO3 31.

What is the patient’s acid-base status?

Uncompensated metabolic acidosis

Primary metabolic acidosis with compensatory respiratory alkalosis

Uncompensated respiratory acidosis

Primary respiratory acidosis with compensatory metabolic alkalosis

Primary metabolic alkalosis with compensatory respiratory acidosis

A

Primary respiratory acidosis with compensatory metabolic alkalosis

Not: The patient has a low pH (acidemia)

• The patient has a high PCO2 (respiratory acidosis) and a high bicarbonate (metabolic alkalosis). The combination of the low pH and the high PCO2 tells us that the respiratory acidosis is the primary process.

• The metabolic alkalosis is the compensatory process. The pH is still low despite this metabolic compensation

Summary: Primary respiratory acidosis with compensatory metabolic alkalosis

237
Q

Which of the following is the preferred intravenous agent of induction of anesthesia for maintaining spontaneous breathing and airway tone

Midazolam
Ketamine
Propofol
Fentanil
Diazepam

A

Ketamine

Ketamine preserves spontaneous respiration and airway tone without causing apnea at induction doses. Propofol, opioids and Feedback: benzodiazepines are associated with respiratory depression at induction doses and cause apnea

238
Q

An 82-year-old man presents to the emergency department after 2 episodes of bright red blood clots in his stool during the past 6 hours. He otherwise feels fine and denies any symptoms including abdominal pain. He has a history of diverticulosis, seen on multiple colonoscopies in the past, and has had a diverticular bleed in the past.

On physical examination, the temperature is 36.5°C, the pulse is 90/min and regular, respirations are 12/min, and blood pressure is 130/86 mmHg. The chest is clear to auscultation, and no murmurs are heard. The abdomen is soft and non-tender, bowel sounds are active.

Results of laboratory studies are pending. Anoscopy shows blood in the rectal vault; no hemorrhoids are noted.

Which of the following is the best next step in management?

Colonoscopy

Exploratory laparotomy

Mesenteric angiography

Administration of fresh frozen plasma

Ultrasound of the mesenteric arteries

A

Colonoscopy

239
Q

A 40-year-old woman comes to the emergency department because of a 2-day history of severe periumbilical, colicky pain, and abdominal distention. She is nauseated and has vomited light green vomitus 6 times in the past 24 hours. She has not passed stools or flatus for 48 hours.
Her medical history is remarkable only for an abdominal hysterectomy 8 years ago.
On physical examination, her temperature is 100.4°F (38°C), her pulse is 110/min and regular, respirations are 24/min, and blood pressure is 120/90 mmHg. No murmurs are heard. The lungs are clear to auscultation and percussion. The abdomen is distended and mildly tender to deep palpation. Bowel sounds are high-pitched and tinkling.
Which of the following is the most likely diagnosis?

Diverticulitis
Viral enteritis
Crohn’s disease
Colon cancer
Adhesive small bowel obstruction

A

Adhesive small bowel obstruction

240
Q

During a left colectomy for left-sided colon cancer, in the retroperitoneum, a firm white band-like tubular structure is seen running cephalad to caudad that shows peristalsis when pinched.

What is this structure?

Psoas tendon

Ureter

Gonadal vein

Common bile duct

Gonadal artery

A

Ureter

Not: The ureter is a muscular structure running in the retroperitoneum from the kidney to the bladder. During colon resections, it is most Feedback: frequently injured on the left when transecting the left or sigmoid colon mesentery. It is found just anterior to the psoas and crosses beneath the gonadal artery before going over the iliac vessels. When gently stimulated, it will show peristalsis

241
Q

In the context of trauma management and resuscitation of the critically injured patient, the proper sequence of the fundamentals of care is which of the following?

Airway control, control of hemorrhage, restoration of perfusion, radiographic imaging

Rapid rewarming, airway control, control of hemorrhage, restoration of perfusion, radiographic imaging

Control of hemorrhage, airway control, restoration of perfusion, rapid rewarming, radiographic imaging

Restoration of perfusion, airway control, control of hemorrhage, rapid rewarming, radiographic imaging

A

Airway control, control of hemorrhage, restoration of perfusion, radiographic imaging

242
Q

A 64-year-old man presents to your office with an incidentally identified abdominal aortic aneurysm (AAA) measuring 5.7cm by screening dyplex examination. He has mild COPD and a history of coronary artery disease, having had coronary stents several years ago after a myocardial infarction. He is able to climb 2 flights of stairs without difficulty. What is the next imaging study that should be obtained?

Arteriography

Computed tomographic angiography (CTA)

Chest X-ray

No imaging is necessary. He should be booked for surgery.

A

Computed tomographic angiography (CTA)

243
Q

A48 year-old male patient with refractory hypertension on four anti-hypertensive medications is found to have Cushing’s syndrome. Review of his medications reveals no exogenous glucocorticoid administration. Which lab result would confirm that this patient is a candidate for laparoscopic adrenalectomy?

High Cortisol

Low ACTH

High ACTH

Mild increase of cortisol in response to administration of corticotrophin releasing hormone

Suppression of cortisol with low-dose dexamethasone

A

Low ACTH

244
Q

Sol böbrek üstü toplardamarı hangi yapıya boşalır?

Alt vena kava

Sol frenik ven

Sol böbrek veni

Sağ böbrek veni

Sol ortak iliak ven

A

Sol böbrek veni

245
Q

A 24-year-old asymptomatic male was found to have a 2.5cm left adrenal mass on CT scan of the abdomen for abdominal pain. He was also found to have appendicitis and underwent laparoscopic appendectomy uneventfully. Upon discharge, you explain to the patient that he has a small adrenal nodule and he is given a follow up appointment. Wish test would NOT be indicated to assess if this mass is a functional adenoma?

Renin/Aldosterone level

Serum electrolyte level

Dexamethasone suppression test

Urinary and serum metanephrine/normetanephrine levels

Plasma Testosterone Level

A

Plasma Testosterone Level

246
Q

A 48-year-old male smoker with no additional significant past medical history is brought to the emergency room after a fall from scaffolding at work. On arrival, he was normotensive at 110/60, P: 80, with temperature of 37. On physical exam, he complained of leg pain on the left and the remainder of his physical exam including neurological evaluation was normal. His PA and lateral chest X-ray revealed no evidence of pneumothorax. He underwent CT scan of the abdomen and pelvis which revealed a 6.2cm left adrenal nodule with no evidence of local Ilyasion. Laboratory results did not demonstrate functionality, What is the next step in management?

Observation

Rule out lung cancer by obtaining CT scan

Open adrenalectomy

Laparoscopic adrenalectomy

CT guided biopsy of adrenal mass

A

Laparoscopic adrenalectomy

247
Q

A 42-year-old male undergoes a CT scan of the chest, abdomen, and pelvis following a motor vehicle crash and is found to have a 3cm left adrenal nodule. What is the most appropriate next step in management?

Repeat CT scan in 3 months

OCT guided core needle biopsy

Laboratory studies with basic metabolic profile, aldosterone, renin, metanephrines and low dose dexamethasone suppression test

Laparoscopic left adrenalectomy

No further work-up is indicated

A

Laboratory studies with basic metabolic profile, aldosterone, renin, metanephrines and low dose dexamethasone suppression test

248
Q

62-year-old male underwent a CT scan for evaluation for right lower quadrant pain to rule out appendicitis. His medical history is significant for hypertension, hypercholesterolemia, and recent myocardial infarction three months ago that required cardiac catheterization and placement of drug eluting stent. His CT scan was unremarkable for appendicitis but incidentally he was found to have 2.5cm adrenal nodule on the right side. Further work up revealed normal serum and urine catecholamine levels, and a plasma aldosterone/renin ratio of 31 and serum potassium level of 3.9. His current medication includes a beta-blocker, ACE-inhibitor, and calcium channel blocker. On physical exam, he was normotensive at 120/80, P: 65 and otherwise in good health. What would you advise this patient at this point?

Adrenal vein sampling after medical optimization

Laparoscopic adrenalectomy for functional adrenal mass

Laparoscopic adrenalectomy for non-functional adrenal mass

Open adrenalectomy for non-functional adrenal mass

Open adrenalectomy for functional mass

A

Laparoscopic adrenalectomy for functional adrenal mass